LE-LR (CR) SEctionals
LE-LR (CR) SEctionals
Reasoning/CR
CLAT - By @cl_mock
Passage(Q.106-Q.110): With the advent of the Covid-19 pandemic, normal life — in India and across the
world — has been fundamentally disrupted. More worrying is the fact that the virus has also adversely
affected life expectancy. A recent study has revealed that life expectancy at birth of both men and women
in India decreased by around two years — from 69.5 years and 72 years in 2019 to 67.5 years and 69.8
years, respectively — in 2020 owing to Covid-19. The term refers to the average number of years that a
new born is expected to live if the mortality pattern at the time of birth remains constant in the future. Given
that the number of Covid deaths in the country has by now crossed 4.5 lakh — this is the official tally —
the drop in life expectancy is not surprising. The pandemic has even undone a decade’s efforts to increase
life expectancy in the country — it has now regressed to the 2010 figures. But this is no reason to lose
heart. Experts argue that such a decline is not uncommon after a health crisis of a massive scale.
The parity in the decline in life expectancy across the gender divide can be misleading: the impact of the
pandemic on men and women has been far from equal. Globally, as in India, more women lost their jobs
during Covid-19 than men. During the lockdown, this difference amounted to 40 percentage points. The
burden of unpaid care work at home, exacerbated by the pandemic, also fell disproportionately on female
shoulders. However, government outreach and palliative measures have not been weighted appropriately
to correct this imbalance — even as India administered one billion jabs, official data indicate that 6 per cent
fewer women are getting vaccinated. Other marginalized constituencies are being affected unevenly too.
The pandemic’s wrath has fallen even more harshly on the transgender community — many have
reportedly been deprived of even the subsistence allowance of Rs 1,500 pledged by the Union government.
The inoculation campaign has, thus, been smeared by existing social cleavages. That the pandemic would
curtail life was expected; but it is the duty of the government to ensure vulnerable constituencies are not
disenfranchised — socially, economically and in terms of health. Vigilance and closer monitoring of the
health and rightful benefits of women and those on the margins are key to equitable healthcare and quality
of life.
[Extracted with edits and revisions from The Telegraph]
108. Which of the following would strengthen the author’s claims in the passage?
(a) Life expectancy in Africa had dropped drastically owing to the HIV/AID epidemic, but it recouped in a
few years.
(b) There is a condition called Lipoma which makes one ineligible for inoculation against Covid 19, this is
more common in women than men.
(c) The Europe till date has not recovered in life-expectancy after the Spanish flu.
(d) Decline in women’s job was at its ebb pre-Covid.
Head Office: 127, Zone II, MP Nagar, Bhopal |+91-7676564400| https://www.toprankers.com Page 29 of 40
109. What can be attributed to be the author’s opinion from the passage?
(a) The impact of Covid 19 on different groups of people was surprisingly not very different.
(b) The lockdown imposed due to Covid 19 was unprecedented and unexpected.
(c) The impact of Covid 19 on different gender groups - men, women and transgender was different.
(d) The Union Government does a better job than state governments in terms of making the society more
welcoming towards vulnerable groups.
110. What does the author mean by this statement, “Government outreach and palliative measures have not
been weighted appropriately to correct this imbalance.”?
(a) The government had the opportunity to correct the imbalance created by Covid 19, but its span in terms
of measures were unevenly distributed, leaving many unimpacted.
(b) The government could not make sure that all the groups are treated equally when the natural floods
took so many lives.
(c) The gender gap was created due to the pandemic and it was the duty of the government to further
increase this divide.
(d) The State governments and the Union government should be held accountable for the gender divide
that they have created.
Passage(Q.111-Q.115): In its most recent meeting, the monetary policy committee of the RBI chose to
maintain the policy status quo, keeping the benchmark repo rate at 4 percent and continuing with its
accommodative stance, presumably as long as is necessary to revive growth. But the decision was not
unanimous. One of MPC’s members voted not only against continuing with the accommodative stance, but
also argued in favour of raising the reverse repo rate. Concerns over how long the MPC can continue with
its policy stance appear to be gaining traction. Even though the other members stuck to their earlier
positions, there appears to be a shift in the tone of the commentary, with another member arguing that
“over-stimulus as after the global financial crisis, with delay resulting in sharp adjustment, has to be
avoided.” So far, the MPC has continued to attach primacy to growth considerations. In their most recent
comments, most members highlighted the uneven nature of the recovery — the scarring of small business
and the informal sector. According to another MPC member, of the 404 industries for which data is
available, 63.4 percent are operating below 2018-19 levels. The RBI Governor also argued that the
“informal sector is likely to take even longer to recoup as the impact of the second wave on this sector was
relatively more pronounced.” But the space for continuing with its current stance appears to be receding.
On the inflation front, the MPC members were cognisant of the risks, as, even though headline inflation
has dipped — CPI fell to 4.35 per cent in September, down from 5.3 per cent in August — core inflation
remains elevated. As the member who voted against continuing with the accommodative stance noted,
“inflationary pressures are beginning to show signs of greater persistence than anticipated earlier”.
Economists expect inflation to inch back upwards. According to the RBI’s projection, it is expected to dip
to 4.5 per cent in the third quarter, rising thereafter to 5.8 per cent in the fourth quarter. The withdrawal of
policy support is likely to be gradual with the process playing out at multiple levels — beginning with
normalisation of liquidity, which has begun, followed by a hike in the reverse repo, a shift in the stance from
accommodative to neutral, culminating in a repo rate hike. However, the duration over which this plays out
is uncertain. Perhaps a few more months of data will provide greater clarity over the durability of the
recovery, allowing the MPC to firm up its exit plans.
Head Office: 127, Zone II, MP Nagar, Bhopal |+91-7676564400| https://www.toprankers.com Page 30 of 40
112. Out of the following, which one is the best representation of the main message of the author in the passage?
(a) It will need greater clarity over durability of recovery for RBI to firm up its moves towards policy
normalisation.
(b) Policy normalisation is a distant goal for the RBI, firstly it needs to bring the members of the MPC to an
agreement.
(c) The informal sector was one of the hardest hit sectors due to the second wave and all efforts must be
made to revive it.
(d) The difference in opinion of the members of the MPC over a subject of particular concern is disturbing,
which might hamper its efficiency.
113. Which of the following can be inferred from the given passage?
(a) Disagreement between members of an organization is a sign of its bleak future.
(b) The RBI is a body which aggravates the already fragile situation of the weaker sectors.
(c) The RBI’s stance on growth considerations is likely to change in future.
(d) The monetary policy committee of the RBI, in the recent meeting, chose to maintain the policy status
quo, keeping the benchmark repo rate at 4 percent.
114. For which of the following claims has the author presented some pieces of evidence?
(a) Till now, the stance of the Monetary Policy Committee on growth considerations has not changed.
(b) The decision of the Monetary Policy Committee to maintain the status quo was not unanimous.
(c) The withdrawal of policy support by the Monetary Policy Committee is likely to be abrupt.
(d) Both a and b
115. ‘In its most recent meeting, the monetary policy committee of the RBI chose to maintain the policy status
quo, keeping the benchmark repo rate at 4 percent and continuing with its accommodative stance,
presumably as long as is necessary to revive growth. But the decision was not unanimous. One of MPC’s
members voted not only against continuing with the accommodative stance, but also argued in favour of
raising the reverse repo rate.’ Which of the following reflects the assumption(s) behind the passage?
I. There are more than one committee that governs the monetary policies of the RBI.
II. The decision to maintain the policy status quo with regard to repo rate was unanimous.
(a) Only assumption I is implied.
(b) Only assumption II is implied.
(c) Both assumptions I and II are implied.
(d) Neither assumptions I nor II is implied.
Passage(Q.116-Q.120): In the current public unrest in France over the rising price of the baguette are
echoes of what happened 232 years ago. Back then, the fact that a staple had been priced out of the reach
of the masses led to the French Revolution and the dramatic, bloody collapse of the ancient régime. The
situation isn’t quite so dire right now — the president of the French Confederation of Bakeries and Pastry
Shops (FCBPS) has said that unlike then, bread is still available, even if it’s more expensive.
Around the world, the unaffordability or non-availability of food has, from time to time, led to mass unrest,
and even the collapse of governments. Bread riots had preceded the fall of the Bastille in 1789, which was
stormed in part because the starving sans culottes were looking for grain. Similarly, in 1918 a precipitous
rise in the price of rice caused riots in Japan, which led to the resignation of Prime Minister Terauchi
Masatake and his cabinet. And consider how frequently that Indian pantry staple — the onion — has caused
electoral convulsions: From fuelling public anger against Indira Gandhi’s government in the pre-Emergency
months to powering her resurgence in the 1980 general elections. Onion price rise was also one of the
factors in the ousting of the BJP in the 1998 Delhi Assembly elections.
France’s current surge in the price of the baguette is due to bad harvests in Russia, which have led to a
global rise in the price of wheat. Greater energy prices have also made ovens more expensive to operate
and the heat is being felt by the nation’s famous boulangeries, as well as the average consumer of the
Head Office: 127, Zone II, MP Nagar, Bhopal |+91-7676564400| https://www.toprankers.com Page 31 of 40
long, baton-shaped bread which is seen as an icon of French culinary heritage. As history teaches us,
when people complain about basic foodstuff being unaffordable, governments can’t just ask them to eat
cake.
[Extracted with edits and revisions from The Indian Express]
116. Which among the following can be deduced from the passage?
(a) The President of FCBPS feels that there is no difference between the food crisis in the past and the
present.
(b) There have been a lot of protests and mass movements against the unavailability or high prices of
exquisite food.
(c) There is no option available to the French Government other than asking its people to eat cake.
(d) None of the above.
117. If the information set out in the passage is true, then which of the following must be true?
(a) There is a link between the high operational costs of Ovens and food prices.
(b) Sometimes a food staple can go out of stock without the fault of the country where the crisis has taken
place.
(c) The non-affordability or non-availability of the food has led to mass riots time and again.
(d) An average consumer can withstand the heat of the food prices or the non-availability of the food by
studying the pattern of food production and consumption.
118. Which of the following can be inferred about the various food crises around the world?
I. Unaffordability of staple diet can lead to mass movements.
II. Unavailability of food can lead to fall of governments.
III. Some leaders of Japan were forced to resign for the unavailability of food.
(a) Only I and II (b) Only II (c) Only I and III (d) All I, II and III
119. What can be inferred about the present food crisis in France?
(a) The situation in France is so grave that there is no food to feed people.
(b) The unavailability of the food concerned is not entirely due to the failure of the French government.
(c) The present food crisis in France is likely to repeat history, with the second French revolution coming
in.
(d) None of the above.
120. ‘As history teaches us, when people complain about basic foodstuff being unaffordable, governments can’t
just ask them to eat cake.’ What can be inferred from the given argument?
(a) The government cannot offer something that is contrary to the expectations of the people in order to
save itself.
(b) History is witness to unrest in various countries as and when the governments have failed to meet even
the basic foodstuff due to inflation.
(c) The history indicates that the wrong alternatives or policies by the government with regard to food prices
and availability led to public unrest and subsequent government fallings.
(d) If people demand the basic foodstuff, the government cannot present exclusive food as an alternative
to appease people in order to avoid ousting.
Head Office: 127, Zone II, MP Nagar, Bhopal |+91-7676564400| https://www.toprankers.com Page 32 of 40
Passage(Q.121-Q.125): Read the passage below and answer the questions that follow:
Humans have always used stories to make sense out of our chaotic world. When our ancestors had to kill
animals they felt were kindred spirits to survive, they created myths to help them come to terms with it.
When they invented agriculture, they created myths that glorified graft and highlighted the seasonal nature
of existence. When they began to settle, humans created myths imbuing cities with transcendence. Fast-
forward to the volatile times we live in today, where people increasingly recognize that tackling problems
like climate change, inequality, and health care requires a systems approach. The work of systems change
involves seeing systemically—looking at the elements, interconnections, and wider purposes of systems—
and acting systemically. Story plays a vital role in helping us do both of these things.
Story has many different qualities that make it useful for the work of systems change. It’s a direct route to
our emotions, and therefore important to decision-making. It creates meanings out of patterns. It coheres
communities. It engenders empathy across difference. It enables the possible to feel probable in ways our
rational minds can’t comprehend. When it comes to changing the values, mindsets, rules, and goals of a
system, story is foundational.
Supporting niches of innovation is another important part of systems change. This not only affects the
elements in a system, but also can transform the nature of the relationships in a system and ultimately its
purpose. A core part of this work involves illuminating outliers: the mavericks, pioneers, and intrapreneurs
who are already trying to make change happen. Story plays two important roles here. First, systems
changers can increase the profile and impact of outliers by amplifying their stories. Second, they can use
story to create unifying narratives of change, adding momentum to innovative movements.
121. Which of the following statements will the author deem the truest?
(a) There are some lessons that cannot be taught without storytelling.
(b) Storytelling cannot be used to provide all kinds of messages and information.
(c) Systemic change can only be brought by storytelling.
(d) Historical evidence points towards the use of storytelling to generalize certain phenomenon to make
human actions make sense.
122. Which of the following conclusions can’t be properly drawn from the statements above?
(a) The work of systems change involves looking at things more wholistically from a wider perspective.
(b) The work done by the change-makers of the society must be exaggerated and presented as stories.
(c) Storytelling is a powerful tool that can be used to harmonize and mobilize people for various causes.
(d) Story is extraneous when trying to change the values, mindsets, rules, and goals of a system
123. Which of the following options can be said to be proper instances mentioned in the passage?
I. Chandler Bing wants to reach the grapes hanging on the high branch of a tree. He tries a lot to climb,
but fails. He ultimately thinks that its better he didn’t eat those grapes, because those grapes are sour.
II. Joey watches a movie where the authority asks his people to make sacrifices of the animals for the
higher good of humanity.
(a) Only I (b) Only II (c) Both I and II (d) Neither I nor II.
124. Which of the following, if true, might weaken the author’s claim?
(a) Various people prefer to have debates over topics rather than monologue discussion
(b) People’s intelligence and analytical ability makes them see through the specious justifications behind
the stories.
(c) Some problems cannot be solved by creating stories revolving around them.
(d) Form the yore, myths have been created to gradually adapt people to system change.
125. The author’s statement that ‘A core part of this work involves illuminating outliers: the mavericks, pioneers,
and intrapreneurs who are already trying to make change happen.’
(a) Forms premise of the above passage.
(b) Forms conclusion of the above passage.
(c) Forms assumption of the above passage.
(d) Forms inference of the above passage.
Head Office: 127, Zone II, MP Nagar, Bhopal |+91-7676564400| https://www.toprankers.com Page 33 of 40
Passage(Q.126-Q.130): Read the passage given below and answer the question that follows.
Indian women excelled in the most decorated Olympic Games for India so far. There is no reason for it to
be otherwise in any other field, especially education, given the right support. As a nation, we can ill-afford
to ignore half the potential workforce if we aspire to be an economic powerhouse. As a society, women can
be the pivot to bring about critical and lasting social transformation. As individuals, they deserve a shot at
being the very best they can.
First, as an immediate step, in every locality, a mohalla school or a community learning programme should
be started with appropriate Covid norms, if the local disaster management authorities and the state
governments permit. Evidence from the Ebola pandemic shows that continued engagement with
educational activities reduces drop-outs in a statistically significant way. NITI Aayog, with the help of civil
society organisations, had started a community programme led by volunteers called “Saksham Bitiya” in
28 aspirational districts where more than 1.87 lakh girl students were trained in socio-emotional and ethical
learning. Such initiatives should be replicated to ensure more girls do not drop out of schools during the
pandemic.
Second, to predict likely drop-outs, a gender atlas comprising indicators that are mapped to key reasons
for school drop-outs should be developed. Teachers should also be trained in all the scholarships and
schemes available that provide economic support to girls and their families for continuing their education.
Third, there is a need to revise the National Scheme of Incentive to Girls for Secondary Education in areas
or states with high prevalence of drop-outs and early child marriages. The scholarship amount may be
increased and tied to the completion of graduation, with yearly scholarships paid to students upon
successful completion of each year of their undergraduate degree.
State governments need to leverage existing schemes to design interventions to promote women in higher
education. The recently modified viability gap funding scheme includes provisions for social infrastructure
projects, including education. For greenfield projects in higher education, 60 per cent of the funding can be
accessed as viability gap funding from the Central and State governments.
The pandemic has brought unprecedented challenges for educators and students, especially for those on
the margins, including girls. However, with recent experiments and learning experience, informed targeting
of ample resources and an agile policy environment, this challenge could well prove to be an opportunity.
Head Office: 127, Zone II, MP Nagar, Bhopal |+91-7676564400| https://www.toprankers.com Page 34 of 40
129. ‘As a society, women can be the pivot to bring about critical and lasting social transformation. As individuals,
they deserve a shot at being the very best they can.’ Which of the following assumption is behind the
argument
(a) Women, individually and socially can bring transformative changes.
(b) Women empower a nation.
(c) A women can help other women in bringing critical and lasting social transformation.
(d) A society cannot survive without women.
130. Being assertive, the author claims that the women have the potential to be the catalyst for
(a) removing the long-lasting stigma of non-education in the society.
(b) a drastic change in the level of incomes and employment generation.
(c) a lasting change in the performance of the assets and shares.
(d) a deep-rooted social change in the communities that we all live in.
Passage B: The trading relationship has allowed a Chinese regime that is antithetical to liberal values at
home and to the existing international system to acquire more power and resources, which it has used to
both pursue greater capabilities to act in the world (often at odds with U.S. preferences), but also to more
effectively repress its citizens at home. Disconnecting the U.S. and Chinese economies, despite the short-
term pain, is ethical in the long run for removing any tacit U.S. support for China’s unliberal practices at
home which are at odds with American values but also to lessen the economic and technological bases
from which China is emerging as a near-peer competitor to the U.S.
133. Both the author of Passage A and the author of Passage B would agree with which one of the following?
(a) A trade war between US and China will benefit US in the long run.
(b) A trade war between US and China will create negative impact on the trades on both sides.
(c) A trade war between US and China will reduce the standard of living of US citizens.
(d) A trade war between US and China can be effectively addressed through discussions.
Head Office: 127, Zone II, MP Nagar, Bhopal |+91-7676564400| https://www.toprankers.com Page 35 of 40
134. Which one of the following most accurately characterizes the author of passage B’s response to the author
of passage A?
(a) The author of passage B falsely accuses the author of passage A of contradicting their own views.
(b) The author of passage B unfairly directs his argument against the author of passage a personally.
(c) The author of passage B uncovers a hidden assumption underlying the author of passage A’s position.
(d) The author of passage B contradicts the reasons the author of passage A cites in favor of her
conclusion.
135. U.S. imposed six waves of tariffs on a number of imports of China. The US’ losses mounted steadily over
the year, as each wave of tariffs affected additional countries and products from washing machines to steel
and aluminum. The US’ losses hit the hardest after the sixth wave, when the U.S. levied $200 billion in
Chinese imports with a 10 percent tariff.
If the above statements are true, based on the reasoning of the author of Passage B:
(a) The loss is justified as China suffered greater loss.
(b) The loss is not justified as it hurt American citizens.
(c) The loss is not justified as it is against liberal practices.
(d) The loss is justified for long-term consideration.
Head Office: 127, Zone II, MP Nagar, Bhopal |+91-7676564400| https://www.toprankers.com Page 36 of 40
103. (d) Option (d) is the correct answer because with the option (a) is the answer. But this is no reason to lose
change of facts, Y cannot be disqualified as he is heart. Experts argue that such a decline is not
merely contesting a municipal election and not uncommon after a health crisis of a massive scale.
elections for becoming Member of Parliament. B is incorrect since it actually adversely affects the
Hence in no case he can be disqualified. Since our claim made by the author that women are less
answer from the previous question is not changing, vaccinated than men. Option (c) is weakening the
the options with ‘yes’ are straightaway eliminated author’s stance, as if the Europe is still reeling with
i.e., A & C and leaving us with B & D; between B & the effect of Spanish flu, then the statement nullifies
D, D is the correct option due to correct explanation. the author’s optimism that life-expectancy post Covid
104. (d) Option (d) is the correct answer because first and will come back to its original state. Option (d) is
foremost, Z was given an honorary position in the contrary to what is mentioned by the author in the
European chambers of commerce and trade not by passage. Refer to the lines, ‘Globally, as in India,
government of India. Hence the concept of office of more women lost their jobs during Covid-19 than
profit will not be applicable in the given case, giving men. During the lockdown, this difference amounted
us D as the best answer. B also seems to be true but to 40 percentage points.’ Therefore, option (d) is
the explanation of D is more comprehensive. Rest of incorrect as it invalidates the author’s claim.
the options (a), (b) and (c) on the basis of the above 109. (c) The correct answer is C. The author has clearly
reasoning are incorrect. talked about the different impacts of the pandemics
105. (a) Option (a) is the correct answer because Z was given on different gender groups. A is incorrect since this
an honorary position by government of India. Lat is in clear contrast with option C, which is correct. B
para of the passage states that came in the case is incorrect because the author has suggested that
of Jaya Bachchan v. Union of India where the court the curtailment of life due to the pandemic was
said that, "payment of honorarium by government of expected. D is incorrect because the author has not
India, in addition to daily allowances like made a comparison between the works of the Union
compensatory allowances, rent-free government and the state governments.
accommodation, and chauffeur-driven car at the 110. (a) The correct answer is A. The government could not
state expense, are clearly in the nature of correct the imbalance by taking appropriate
remuneration and a source of pecuniary gain and measures to mitigate the gender divide. The
hence constitute office of profit. Hence the concept measure did not take into consideration certain
of office of profit will be applicable in the given case. sections that got left behind. This is what the author
Since our answer from the previous question is means by the given statement. Therefore, option (a)
changing, the options with ‘No’ are straightaway is the correct option. B is clearly incorrect since
eliminated i.e., (b) & (d) and leaving us with (a) & (c); natural floods are out of context. C is incorrect
between (a) & (c), (a) is the correct option due to because the duty of the government was not to
correct explanation. increase the gender gap, it should take steps to
reduce the gap. D is incorrect because the
SECTION - D : LOGICAL REASONING governments did not create the gender gap, the
pandemic did.
106. (b) The correct answer is B. The author does 111. (c) The correct answer is C. There were some members
acknowledge the dip in life expectancy but is hopeful of the MPC who voted against maintaining the status
that it would recover in a few years and is more quo. Therefore, this option is true according to the
concerned about the impact of the pandemic on passage. A is incorrect since the author has clearly
different constituencies, i.e., different groups of stated that there is some uncertainty over the
people. A is incorrect since the author is not much duration of the withdrawal of the policy support. B is
concerned with the dip in life expectancy. C is incorrect since this is in clear contrast with Option C.
incorrect since this is not the main idea of the D is incorrect since the author has indicated that the
passage. Whether the author was surprised at this fall in CPI indicates fall in the inflation rate.
fact is not sufficiently relevant in this passage to be 112. (a) The correct answer is A. This is the best
considered the central concern of the passage. D is representation of the main idea of the passage. The
incorrect because the author does not blame the RBI is uncertain over its plans to withdraw the policy
government about the pandemic. support and it needs clarity over the durability of
107. (d) The correct answer is D. I and II are incorrect recovery to make a firm move on the policy support
because the data given is about average life that it provides. B is incorrect since the second part
expectancy and not minimum life expectancy. III is of the option is not of any concern of the passage. C
incorrect because the decline in life expectancy in is incorrect since this is not the main concern of the
women is sharper than that of women. Therefore, D passage but only a general piece of information. D is
is correct. incorrect as well, for the same reason as that of
108. (a) The correct answer is A. A clearly strengthens the Option B’s second part.
claims of the author made in the last part of the first 113. (c) The correct answer is C. The author has indicated
paragraph. Refer to the lines, ‘The pandemic has that the RBI’s stance on growth considerations is
even undone a decade’s efforts to increase life likely to change with time as there is more clarity over
expectancy in the country — it has now regressed to the durability. Refer to the lines, ‘The withdrawal of
the 2010 figures.’ The fact that life-expectancy policy support is likely to be gradual with the process
regained in Africa owing to the HIV/AID epidemic is playing out at multiple levels — beginning with
on similar lines with that of the author’s. Therefore, normalisation of liquidity, which has begun, followed
Head Office: 127, Zone II, MP Nagar, Bhopal |+91-7676564400| https://www.toprankers.com Page 12 of 16
by a hike in the reverse repo, a shift in the stance considers the incorrect statement III. Option (d) is
from accommodative to neutral, culminating in a incorrect for the same reason as option (c).
repo rate hike. However, the duration over which this 119. (d) The correct answer is D. A is incorrect because this
plays out is uncertain. Perhaps a few more months is clearly in contrast with the author’s opinion. The
of data will provide greater clarity over the author does not mention that the situation is grave;
durability of the recovery, allowing the MPC to in fact, it is under control. Also, the matter is about
firm up its exit plans.’ A is incorrect since this is not the rise in the price of the baguette. B is incorrect
of any concern in the passage. B is in clear contrast because the present food crisis in France is due to
with what the pieces of evidence in the passage unaffordability of food, not unavailability of food. C is
suggest. Option (d) is incorrect, as it is explicitly incorrect because the author has not presented any
mentioned in the passage. pieces of evidence to support this statement. Since
114. (d) The correct answer is D. The author has presented none of the options are correct, option (d) is the
evidence for option A in the subsequent sentence of answer.
“So far, the MPC has continued to attach primacy to 120. (c) The main statement is the conclusion of the author
growth considerations.” The author has presented with regard to the passage. Instead of taking the
some pieces of evidence for option B in the statement in isolation, it needs to be understood in
subsequent sentence of “But the decision was not totality. The author in the passage cites many
unanimous.” Option C is a claim which is examples of how not being able to provide basic
inconsistent with the author’s claims. foodstuff has led to unrest amongst people, at the
115. (d) Neither of the assumptions are inbuilt in the cost of the incumbent governments’ getting ousted.
passage. Assumption I, though partly true in terms of Therefore, the last line in the passage serves as the
that there are more than one committee of the RBI, author’s contention that the government cannot
since the passage mentions the monetary policy present alternatives in place of demands made by
committee of the RBI, it is based on the assumption the public. Such a thing in the past has costs the
that the author must put up the name of the governments dearly. Option (c) best reflects the
committee in order to avoid confusion, but the latter inference. Option (a), though close, cannot be the
part of the statement cannot be supposed apart from answer as it is vague in terms of expectations (what
the monetary policy committee, the other expectations?). Option (b) is very specific, with
committees also look into the monetary aspect. inflation being cited as the only factor. Option (d) is
Therefore, assumption I is incorrect. Assumption II is taken in the literal sense; therefore, does not make
incorrect as it nullifies the argument completely. The sense.
argument clearly states that one member was not in 121. (d) Rationale: Option (A) is incorrect as it cannot be
unison to the decision. Therefore, assumption II is interpreted from the passage. Nowhere, does the
incorrect, which makes option (d) as the answer. author say that people need to resort to story-telling
116. (d) The correct answer is D. A is incorrect since the to teach all the things in life.
President of FCBPS believes that for now there is Options (B) is an extreme statement which can’t be
availability of food, but in the past there wasn’t any inferred from the passage.
availability of food. B is incorrect. According to the Option (C) again is an extreme statement. While the
passage, mass protests have been done primarily author does mention the importance of story-telling
against unaffordability or unavailability of food of the in systems change, he never says it to be the only
masses. C is incorrect, as this is clearly what the means to bring systemic changes.
author is against, as is clear from the last sentence Option (D) is correct. The author has mentioned in
of the passage. the first paragraph and has started with examples to
117. (b) The correct answer is B. Option (a) is incorrect as support this claim. Refer to the line, ‘Humans have
there is no direct link between the high operational always used stories to make sense out of our chaotic
costs of Ovens and food prices. Fluctuations in the world.’
food prices are matters of demand and supply; Therefore, the correct answer is Option (D).
whereas the operational costs of ovens is because 122. (d) Rationale: Option A can be concluded as it the basic
of greater energy price. Option B must be correct, working of the Systematic change. Refer to the lines,
since this time in France, without any direct fault of ‘The work of systems change involves seeing
France or its government, the prices of bread went systemically—looking at the elements,
up. Option C is a direct extraction from the passage; interconnections, and wider purposes of systems—
therefore, does not add anything further to the line of and acting systemically.’
thought, making it an incorrect option. Option D is Option B can be inferred as the author says that the
far-fetched, even as a furtherance, making it the works of the outliers must be amplified to inspire and
flawed option. bring changes among the people.
118. (a) The correct answer is A. I and II are clearly Option C is a general statement that the author has
mentioned in the first sentence of the second mentioned enough times.
paragraph. However, III is incorrect because it was Option D cannot be drawn as it is contrary to what
the unaffordability and not unavailability of food that has been mentioned in the passage. Story is not
led to riots and subsequently resignation of the extraneous, but foundational.
leaders of Japan. Therefore, option (a) is the correct 123. (c) Rationale: Option (A) is correct as in the first
answer. Option (b) is incorrect, as it does not take paragraph the author says that people fabricate
into account statement I. Option (c) is incorrect as it stories as per their activities and to justify the
Head Office: 127, Zone II, MP Nagar, Bhopal |+91-7676564400| https://www.toprankers.com Page 13 of 16
happening around them. Thus, we see the example contain hence option (A), (B), and (C) are not
of that in this option. relevant, making option (D) a correct answer.
Option (B) is correct as the author says that stories 129. (a) An assumption is the basis of an argument. It is the
connect to people emotionally. Refer to the lines, core idea behind the passage. Option (a) serves as
‘When our ancestors had to kill animals that they felt the basis of the argument. Option (b) is not the
were kindred spirits to survive, they created myths to foundation on which the given argument can stand.
help them come to terms with it.’ Option (c) is far-fetched as the assumption behind
Thus, option (C) is the answer where both the the argument, as the argument does not mention
options are correct. that a women can help the other women. Option (d)
124. (b) Rationale: Option (A) does not affect the stance take is equally far-fetched as the assumption behind the
the author and is incorrect. It is in fact just a general passage.
statement. It has no effect whatsoever on the 130. (d) In the words of the author, “As a society, women can
author’s claims. be the pivot to bring about critical and lasting social
Option is (B) weakens the stance taken by the author transformation. As individuals, they deserve a shot
by attacking the very core of the argument. If people at being the very best they can.” So, it can be said
are intelligent enough to see through the that only women have the capability to be the
justifications, then it becomes difficult to feed them catalyst for the deep-rooted transformation in our
any story to make systematic changes. Therefore, society. So, the correct answer will be option (D).
option (b) is the answer. While, the rest of the options may be true as
Option (c) is vague and illogical. It is irrelevant to the collaterals, they do not specifically find any mention
argument. in the passage. Also, all other options are extreme in
Option (D) strengthens the claims of the author, that terms of reasoning, rendering options (a), (b) and (c)
from time immemorial, stories have been used to incorrect.
glorify the actions of humans and gradually 131. (c) The author of passage A argues that a trade war
conditioning humans through systematic changes. instigates conflict which is regarded as unethical. So,
Therefore, the correct answer is option (B). the principle (or the assumption) here is that we
125. (a) Rationale: Premises are the facts or evidence that should avoid conflict and promote free trade. That is
support or lead to the conclusion. Therefore, the something that is ethical. This is captured in answer
above statement forms premise of the passage choice (c).
which leads to the conclusion. Hence, option (A) is Incorrect Answers
the correct answer. The rest of the options do not fit, (a) and (d) - The issue whether values are aligned
as the main statement is not a conclusion, but an is not something that the author of Passage A
example to lead to the conclusion; therefore, option addresses. So, answer choices (a) and (d) are
(b) is ruled out. The statement also does not form the incorrect.
basis of any argument; therefore, is not an (b) - The author of Passage A argues that trade war
assumption, making option (c) incorrect. It is also not negatively impacts standard of living and so, it
something that one can derive at, as it is an example should be avoided. Avoiding such a thing
given by the author, therefore, option (d) is incorrect. therefore is ethical. Answer choice (b) holds a
126. (a) The author has suggested various remedies to contrary view stating it is unethical.
prevent drop-outs among girls, especially in the field 132. (d) The author of Passage B argues that Chinese
of education mainly during Covid times, as women regime has used trading relationship to promote
can contribute significantly to make the country policies that are against liberal values that US holds.
economically robust. He suggested that the The principle (or assumption) here is that if an entity
scholarship amount may be increased for helping is antithetical to liberal values, then we should avoid
girls with their education. Option (a) is correct as cooperation which such an entity. This is captured in
‘prescriptive’ means to prescribe a treatment for any answer choice (d).
ailment. Option (b) is incorrect as the passage is not Incorrect Answers
just a description of some event or incident or any (a) - This is in direct contradiction with the views
fact. The passage is not an abstract by going in some held by the author of Passage B. For author of
philosophical mode, making option (c) incorrect. The the Passage B, encouraging cooperation with
above passage is also not any event described in the those who have conflicting values would be
form of an anecdote, making option (d) incorrect. unethical.
127. (b) Option (B) suggests that the author is emphasizing (b) and (c) - standard of living and avoiding conflict
that the girls do not drop out of school by providing are not addressed by the author of Passage B.
solutions, making option (B) correct. Option (A) is These are things that are addressed by the
also not the actual intent behind this passage. author of Passage A.
Propagating economic support is one part of the 133. (b) Author of Passage A has clearly stated that the trade
story, eliminating option (C). Also, the author has not war would create conflict. Author of Passage B has
warned anyone, so option (D) is clearly eliminated. clearly stated that there could be short-term pain; it
128. (d) As per the author of the passage-“the recently means that both sides are likely to be hit by the trade
modified viability gap funding scheme includes war, even if for a short period of time.
provisions for social infrastructure projects, including Incorrect Answers
education.” Nowhere in the passage does the author (a) - Author of Passage A will disagree with this
has mentioned anything that a funding plan does not statement while author of Passage B will agree
with this statement.
Head Office: 127, Zone II, MP Nagar, Bhopal |+91-7676564400| https://www.toprankers.com Page 14 of 16
(c) - This is addressed only by author of Passage A. 1400 5 8
= 100 [9 × 18 + 13 × 26] = 14 × 26 = 364
The author of Passage B does not discuss
Girls playing Cricket, Hockey and Chess together
standard of living of US citizens. Hence, we 1400 1 4 1
cannot say for certain whether the author of = [ × 20 + × 18 + × 16] = 14 × 20 = 280
100 5 9 2
Passage B will agree or disagree with the Required difference = 364 − 280 = 84
statement. Hint [141-145]: Common Explanation:
(d) - Discussions between the two countries is not Let students in Mechanical, Electrical and
addressed by either of the authors. Electronics be 12x, 16x and 5x respectively
134. (d) Answer choice (d) is correct as the author of passage 2
Students in IT = ( × 12𝑥) − 140 = 8𝑥 − 140
B negates the reasons that are presented by Author 3
A who is supportive of the US- China trade Students in Civil= 8𝑥 − 140 + 5𝑥 + 30 = 13𝑥 −
relationship and believes that the conflict in trade can 110
be detrimental to both in the long run. ATQ, 12𝑥 + 16𝑥 + 5𝑥 + 8𝑥 − 140 + 13𝑥 −
Incorrect Answers . 110 = 2450
(a) – The option is incorrect as the author of passage 54𝑥 − 250 = 2450
B does not accuse the author of passage A of 𝑥 = 50
contradicting their own views, whether So, Number of students in Civil= 540
700
accurately or falsely. Boys in mechanical = × 540 = 420
900
(b) - The option is incorrect as the author of passage Let Boys in Electrical and Civil be 3y and 2y
B as the author is not getting personal with the respectively
author of passage A. 75 3𝑦
Boys in IT = × 2𝑦 =
(c) – There is no hidden assumption underlying the 200 4
author of passage A’s contention. The author of Boys in Electronics = 3y—210
3𝑦
passage B simply negates the reasoning of ATQ, 3𝑦 + 2𝑦 + 3𝑦 − 210 + 420 + 4 = 1260
author of passage A by putting up contradictory 35𝑦
=1050
views. 4
135. (d) The author of Passage B claims that even though y= 120
disconnecting Chinese and US economies could Departments Total Boys Girls
have short term pain, it is ethical in the long term. So, Mechanical 600 420 180
the author of Passage B will consider these negative Electrical 800 360 440
impacts as short term pains. This makes answer IT 260 90 170
choice (d) the correct answer. Electronics 250 150 100
Incorrect Answers Civil 540 240 300
(a) - There is nothing in the statements that suggests 141. (a) Total number of streams in which girls are more
that China suffered loss. than boys= 3
(b) and (c) - The author would consider the action as 142. (d) Required difference= 600 − 350 = 250
justifiable, as they are in sync with the author of 420+240+150
143. (b) Required average = = 270
passage B. 3
144. (c) Number of students shifted from mechanical to IT =
20
× 600 = 120
100
SECTION - E : QUANTITATIVE TECHNIQUES Number of girls shifted from mechanical to IT=
(250 − 170) = 80
136. (b) Boys playing Cricket and Baseball together Number of boys shifted from mechanical to IT=
1400 4 11 1400
= 100 [5 × 20 + 16 × 8] = 100 × 21.5 = 301 120 − 80 = 40
Girls playing Baseball and Badminton together Boys left in mechanical after shifting = 420 − 40 =
1400 5 5 1400 380
= 100 [16 × 8 + 13 × 26] = 100 × 12.5 = 175 250
301
145. (d) Required percentage = × 100 = 21%
1190
Required Ratio = = 43 ∶ 25 Hint [146-150]:
175
137. (d) Total boys playing Baseball, Badminton and Common Explanation:
Hockey together People who like Pepsi= 350
1400 11 8 5
= [ × 8 + × 26 + × 18] = 14 × 31.5 = 441 People who like only Pepsi= 165
100 16 13 9
Total girls playing Cricket, Hockey and Chess People who like all three soft drinks together
1
together = × 165 = 33
5
1400 1 4 1 5
= 100 [5 × 20 + 9 × 18 + 2 × 16] = 14 × 20 = 280 People who like Fanta = 7 × 350 = 250
Required difference = 441 − 280 = 161 8
People who like only Limca = 3 × 33 = 88
1400 1 7
138. (c) Required average = 100 × 4 [16 + 10 + 8 + 8] = 2 × Let people who like only Fanta and Limca together
42 = 147 be 𝑥
139. (a) Total no. of girls playing all games together Then, people who like only Pepsi and Limca
1400 1 5 5 4 1
= 100 [5 × 20 + 16 × 8 + 13 × 26 + 9 × 18 + 2 × 16 + together = 𝑥 + 20
1 People who like only Fanta and Pepsi together =
× 12] = 14 × 36.5 = 511 𝑥 − 10
3
140. (b) Boys playing Hockey and Badminton together ATQ, 165 + 𝑥 + 20 + 33 + 𝑥 − 10 = 350
𝑥 = 71
Head Office: 127, Zone II, MP Nagar, Bhopal |+91-7676564400| https://www.toprankers.com Page 15 of 16
SECTION - D: LOGICAL REASONING
Directions (Q.106-Q.135): Read the passage carefully and answer the questions that follow.
Passage (Q.106-Q.108): There is a growing feeling, among those who have the responsibility of managing
large economies, that the discipline of economics is no longer fit for purpose. It is beginning to look like a
science designed to solve problems that no longer exist. A good example is the obsession with inflation.
Economists still teach their students that the primary economic role of government - many would insist, its
only really proper economic role - is to guarantee price stability. We must be constantly vigilant over the
dangers of inflation. For governments to simply print money is, therefore, inherently sinful. If, however,
inflation is kept at bay through the coordinated action of government and central bankers, the market should
find its "natural rate of unemployment," and investors, taking advantage of clear price signals, should be
able to ensure healthy growth. These assumptions came with the monetarism of the 1980s, the idea that
government should restrict itself to managing the money supply, and by the 1990s had come to be accepted
as such elementary common sense that pretty much all political debate had to set out from a ritual
acknowledgment of the perils of government spending.
This continues to be the case, despite the fact that, since the 2008 recession, central banks have been
printing money frantically in an attempt to create inflation and compel the rich to do something useful with
their money, and have been largely unsuccessful in both endeavours. We now live in a different economic
universe than we did before the crash. Yet the language of public debate, and the wisdom conveyed in
economic text books, remain almost entirely unchanged.
One expects a certain institutional lag. Mainstream economists nowadays might not be particularly good at
predicting financial crashes, facilitating general prosperity, or coming up with models for preventing climate
change, but when it comes to establishing themselves in positions of intellectual authority, unaffected by
such failings, their success is unparalleled. To this day, economics continues to be taught not as a story of
arguments but rather as something more like physics, the gradual realization of universal, unimpeachable
mathematical truths.
106. Which of the following most accurately expresses the main point of the second paragraph of the passage?
(a) The 2008 financial crisis has affected our financial systems so severely that we cannot do anything to
make it right.
(b) The 2008 financial crisis has changed many concepts which we assumed and, therefore, we must
evolve in our understanding of economics.
(c) The 2008 financial crisis made us print a lot of money in an attempt to force the rich to use their money,
but it did not work.
(d) The 2008 financial crisis should be taught in our textbooks as a reminder to what we could have done
differently.
108. Which of the following is the author most likely to agree with?
(a) Governments cannot print excess money as it would lead to inflation.
(b) Today, economists are successfully predicting financial crashes and preventing climate change.
(c) Banks have printed extra money in the past to induce inflation in the markets artificially.
(d) Economists are moving towards the mathematical aspect of economics as it provides more promising
results.
Head Office: 127, Zone II, MP Nagar, Bhopal |+91-7676564400| https://www.toprankers.com Page 27 of 36
Passage (Q.109-Q.113): The deliberate or unintentional spread of misinformation, despite capturing
widespread public attention, remains as rampant as ever, showing up recently in the form of false claims
about COVID-19 vaccines, the Capitol riot, and many other instances. This “infodemic” is polarizing politics,
endangering communities, weakening institutions, and leaving people unsure what to believe or whom to
trust. It threatens the foundations of democratic governance, social cohesion, national security, and public
health.
Misinformation is a long-standing problem that demands long-term, sustainable solutions as well as short-
term interventions. We've seen a number of quicker, technological fixes that improve the social media
platforms that supply information. Companies like Facebook and Twitter, for example, have adjusted their
algorithms or called out problematic content. We've also seen slower, human-centred approaches that
make people smarter about the media they demand to access online. Evidence-driven educational
programs, for instance, have made people better at discerning the reliability of information sources,
distinguishing facts from opinions, resisting emotional manipulation, and being good digital citizens.
It hasn't been enough. If we're to stop misinformation and its insidious effects, we need to radically expand
and accelerate our counterattacks. It will take all sectors of society: business, non-profits, advocacy
organizations, philanthropists, researchers, governments, and more. We also need to balance our efforts.
For too long, too many resources and debates have focused on changing the technology, not educating
people. This emphasis on the supply side of the problem without a similar investment in the demand side
may be a less effective use of time and energy and clearly, it has been the opposite of the ideal steps that
needed to be taken. While technology-centered, self-policing solutions—filtering software, artificial
intelligence, modified algorithms, and content labeling—do have the ability to make changes quickly and
at scale, they face significant ethical, financial, logistical, and legal constraints.
109. Which of the following statements will the author deem true?
(a) The spread of misinformation is not a recent problem that has cropped up in the world.
(b) Amidst the huge amount of negative news that’s emanating in the society today some positive
misinformation is good for us as well, especially during the pandemic times.
(c) Institutions that supply information can themselves solve the problems with certain steps. Only the
intent from these institutions is needed. It has been lacking amongst them. Their steps haven’t been
enough.
(d) None of the above.
110. Which of the following, if true, weakens the author's claim?
(a) A startling research observation has been that 70% of the information available on the internet today is
misinformation.
(b) It has been observed by a large number of studies that major corporations distribute more information
than individuals.
(c) International laws do not prohibit spreading misinformation by citizens of another nation.
(d) All people are against spreading and receiving misinformation, yet they are forced to get it because of
careless policies of organizations supplying them with such false facts.
111. Consider the following instances –
1. Tony Stark receives a message from Steve Rogers (a prankster, by the way) that the earth is a flat
surface and the people who call it round are mentally unstable. There are also some supposedly "true
scientific facts" to support this claim. Tony, in his innocence, believes this and forwards the message
unknowingly.
2. Albert is a pedant and a researcher who devours knowledge. He gets fascinated by Facebook and
Twitter. After some time, though, he gets frustrated with misinformation and quits social media
altogether.
3. The new sound space social media Addabaaz has become a field-space for misinformation with the
celebrity Angana Raha, spreading misinformation and hate. Addahouse changes their policy that
whenever such instances occur, and they receive complaints from the users, they shall remove the
problematic content.
Which of the above instances can be said to be examples of what has been mentioned in the passage?
(a) Both 1 and 2 (b) Both 2 and 3
(c) Both 1 and 3 (d) All 1, 2 and 3
Head Office: 127, Zone II, MP Nagar, Bhopal |+91-7676564400| https://www.toprankers.com Page 28 of 36
112. Who should have the most significant responsibility in taking steps to immediately stop the menace of
misinformation out of the following?
(a) The ordinary people, because they are the victims of the misinformation.
(b) The companies, because they are technologically much more advanced than the means available to
counter the spread of misinformation, plus the misinformation is happening on their platforms, so they
are in a better position to stop this.
(c) The political class, because they can educate the common folks in a much better way than anyone
else.
(d) All of the above have equal responsibility.
113. The statement "Clampdown on information providing organizations is not enough" is:
(a) Probably True (b) Definitely True
(c) Probably False (d) Definitely False
Passage (Q.114-Q.118): A lot of us think that the World Wide Web and the internet are the same, but they
aren’t. The Web is the most popular way to access online data through hyperlinks and websites; while the
internet, is a vast network of computers and servers on which the World Wide Web operates. The internet
was a tool for scientists, engineers and the military; the web made it accessible to everyone else.
Berners-Lee worked at CERN, where he developed the very first webpage; it went live in August 1991, is
still active, and is probably the world’s first website (bit.ly/2SnA7zy). It was in 1994, however, that the World
Wide Web Consortium (W3C) founded by Tim Berners-Lee set up protocols, guidelines and standards for
the web, and now-familiar terms like TCP (Transmission Control Protocol), IP (Internet Protocol) and HTTP
(Hypertext Transfer Protocol) were born.
The founding philosophy of the Web was for it to serve as a democratizer and equalizer, to empower the
long tail and eliminate monopolies and intermediaries. The Web did solve three big problems for us: an
information problem with search and wikis, a communication problem with email and messenger tools, and
a distribution problem with file-sharing and e-commerce. But it could not address the two big problems that
it was supposed to solve: one of trust and security, and another of disintermediation—its original
philosophy. In fact, the rise of big tech companies has given us intermediaries that are far more powerful
than ever before. They literally own most of our online data and information. In that sense, they own us.
Now comes the Block chain technology.
The reason for the excitement around blockchain is that it is supposed to solve our unsolved problems—
of trust and of inequality—and thereby bring us closer to the original vision of Time Berners-Lee and his
co-conspirators. So, it is not surprising what he said in the Financial Times, as he announced that Sotheby’s
would auction off the original source code of the Web: The NFT project was his “first foray into crypto", but
he saw similarities in his original vision for the web and the philosophy behind the decentralized network
of Ethereum’s blockchain, which underpins most NFTs. It also resonates with his latest project, Solid, which
is designed to give us back control of our personal data. “The blockchain and Solid communities share the
motivations of wanting to empower people," he said, adding that blockchain projects were motivated by
resistance to central control. Much like the open, democratic and decentralized origins of the Web.
Head Office: 127, Zone II, MP Nagar, Bhopal |+91-7676564400| https://www.toprankers.com Page 29 of 36
114. Which of the following conveys the core message discussed in the passage?
(a) The founding of blockchain technology and its application.
(b) The differences and similarities between internet and the World Wide Web and their contribution to the
world.
(c) The original vision of web and its later deviation, only to be brought closer through block chain
technology.
(d) The limitation of web and its failure in providing immunization against trust and security and
disintermediation.
115. Which of the following is true about the World Wide Web?
(a) The web was invented to act as a democratizer and solve the issue of dearth of information.
(b) The web is yet to solve the communication and the information problem.
(c) The web solved the issue of intermediation as per its original philosophy.
(d) The author corroborates the perception of the public that World Wide Web and internet are the same.
116. What does the author mean by stating “They literally own most of our online data and information. In that
sense, they own us.”
(a) Our data is our identity and those who own our data, own us.
(b) Data is the modern currency of exchange in the tech riddled world.
(c) The big companies do not care about data but about us, the users.
(d) Data is the collection of new information and is the costliest on earth.
117. In the last paragraph, the author ends the passage on a __________ note.
(a) Dismal (b) Hopeful (c) Pensive (d) Confrontational
118. According to the passage, why Berners-Lee is excited about the network of Ethereum?
(a) Because the work is a complete imitation of his work.
(b) Because he will be well paid by Ethereum.
(c) Because his philosophy behind creating web matches with Ethereum’s technology.
(d) Because of the centralized network of Ethereum’s blockchain.
Passage (Q.119-Q.123): The French mathematician Pierre-Simon Laplace (1749-1827) believed that the
Universe was a piece of machinery, and that physics determines everything. Napoleon, who had read up
on Laplace's work, confronted him about the conspicuous absence of a creator in his theory. 'I had no need
of that hypothesis,' came the reply. Laplace might have said the same thing about free will, which his
mechanistic universe rendered superfluous.
Since Laplace's day, scientists, philosophers and even neuroscientists have followed his lead in denying
the possibility of free will. This reflects a widespread belief among theoretical physicists that if you know
the initial values of the variables that characterise a physical system, together with the equations that
explain how these variables change over time, then you can calculate the state of the system at all later
times. For example, if you know the positions and velocities of all the particles that make up a gas in a
container, you can determine the positions and velocities of all those particles at all later times. This means
that there should be no freedom for any deviation from this physically determined trajectory.
Consider, then, that everything we see around us - rocks and planets, frogs and trees, your body and brain
- is made up of nothing but protons, electrons and neutrons put together in very complex ways. In the case
of your body, they make many kinds of cells; in turn, these cells make tissues, such as muscle and skin;
these tissues make systems, such as the heart, lungs and brain; and these systems make the body as a
whole. It might seem that everything that's happening at the higher, 'emergent' levels should be uniquely
determined by the physics operating beneath them. This would mean that the thoughts you're having at
this very moment were predetermined at the start of the Universe, based on the values of the particle
physics variables at that time.
Head Office: 127, Zone II, MP Nagar, Bhopal |+91-7676564400| https://www.toprankers.com Page 30 of 36
119. Which of the following statements may have formed the premise for Laplace's argument that 'I had no need
of that hypothesis'?
(a) He was an atheist and did not believe in the existence of a sole creator, after all.
(b) He proved that biological activity at the micro level is literally grounded in the physical shape of
biological molecules.
(c) He might have proved or formulated the existence of all events in the Universe on the basis of physics
and mathematical equations, rejecting the plausibility of free will.
(d) He failed to explain some of the events which were beyond the realms and reach of theories of physics.
120. Which of the following would undermine the argument made in the passage that if you know the initial
values of the variables that characterise a physical system, together with the equations that explain how
these variables change over time, then you can calculate the state of the system at all later times?
(a) Ions are atoms that have become electrically charged because they have lost or gained an electron.
(b) Heisenberg's uncertainty principle introduces an unavoidable fuzziness and an irreducible uncertainty
in quantum outcomes. You might know the value of one variable, such as a particle's momentum; that
means you can't accurately detect another, such as its position.
(c) An apple is suspended from a branch of the tree by a string attached to its stalk. It would thereby have
been turned into a pendulum, because the string constrained its motion. Instead of dropping to the
ground, it would have swung back and forth in a circular arc under the branch, with its state of motion
determined uniquely by its initial position and velocity.
(d) All of the above.
121. Which of the following can we infer from the passage above?
(a) The state of a system is described by what's known as its wave function, which determines the
probabilities of different outcomes when events take place.
(b) At very small scales, theory of Physics underlies what's happening in the world.
(c) The structure of the molecules is truly the secret of life.
(d) Outcomes don't depend only on the equations and the initial data.
122. Which of the following is most likely to be true had theories of physics and mathematical equations failed
to explain some of the earthly events?
(a) Learning and memory offer example of how downward causal effect shapes the underlying physics.
(b) If you seriously believe that fundamental forces leave no space for free will, then it's impossible for us
to genuinely make choices as moral beings.
(c) The power of choice enables physiological systems such as the heart and brain to function in a way
that is enslaved by the lower-level interactions, and choosing the outcomes of the preferred interactions
from a multitude of options have been predetermined at the elementary level.
(d) None of the above.
123. Which of the following most accurately expresses the main point of the passage?
(a) The confounding thing for free-will sceptics is that all outcomes don't depend only on the equations and
the initial data; they also depend on randomization.
(b) In a physically determined world, there is freedom for deviation up to a certain extent.
(c) There are events or things which Physics is unable to answer or reach to an acceptable conclusion.
(d) An understanding of physics sees determinism at work in the Universe.
Head Office: 127, Zone II, MP Nagar, Bhopal |+91-7676564400| https://www.toprankers.com Page 31 of 36
Passage (Q.124-Q.128): Some of us think. Many of us think that we think. And most of us never think of
thinking. Very few use the grand prerogative of the mind, many never think, but they think they do. Very
few of us really think because, to think, we should have the freedom to do so.
We touch the earth with our physical foot; we touch life with our psychological foot. That is why in temples,
we leave our slippers outside. We leave certain coverings that cover our feet and enter the temple barefoot.
In the nakedness, you discover the truth. Shoes cover our feet, and thereby we don’t deeply connect to
God. Our psychological foot, too, is covered by psychological shoes, which are our conditioning, dogmas,
opinions, and beliefs. With the psychological shoes on, we never touch a life.
Very few of us can look at a flower as it is. When somebody looks at a flower, they look at it with the
psychology of likes and dislikes, of should or should not, or must or must not. Therefore, very few of us
really look at the flower; instead, we look at our opinion of the flower- our likes and dislikes.
So, we never see it as it is. Removing the shoe means removing the psychological shoes of our
conditioning, our opinions, and dogmas. When you leave these and innocently look at life, it is then you
see that something very different happens. If you analyze it, you will see that if we are unhappy in life, it is
not because of what is; it is because of our opinion of how life should be. When our view of life conflicts
with our life, this makes us unhappy. This unhappiness is a toxin, a disturbance in our energy field. You
look at situations that you feel should not be, and you get angry. Your anger is often directed as a
scapegoat, not the real genesis of your unhappiness.
124. What does the author talk about when he says ‘the grand prerogative of the mind’?
(a) The illusion of thinking while not thinking in reality.
(b) Utilizing the power of mind to think freely.
(c) Not having the freedom to think.
(d) Actually thinking something/anything at all
125. Why has the author mentioned the psychological shoe?
1. To show that it is deceptive.
2. To show how it covers are true understanding of the world.
3. To show how it distorts our thinking.
(a) Only 1 (b) 1 and 2 (c) 2 and 3 (d) All of the above
126. Based on the information above, which is the most accurate?
(a) We are unhappy because we gauge anger from the wrong prism.
(b) To have an objective clarity about happiness, unhappiness and anything in life, the opinionated shoe
needs to be left out.
(c) The real source of unhappiness and anger lies within us.
(d) The naked soul and the bare feet help connect well with the world.
127. It can be inferred from the passage that the reason of our unhappiness is-
(a) Our opinion is only reason responsible for lack of happiness in our lives.
(b) Consonance in what we think happiness is and what happiness in reality is.
(c) Our psychological shoe, that prevents us from connecting and analyzing things in their natural colour.
(d) Differentiated opinions on life.
128. Statement: ‘Therefore, we never see it as it is.’
The above statement is:
(a) Conclusion statement. (b) Weakening statement.
(c) Strengthening statement. (d) Parallel reasoning.
Head Office: 127, Zone II, MP Nagar, Bhopal |+91-7676564400| https://www.toprankers.com Page 32 of 36
129. In the next few years, molecular biologists will complete mapping the human genome. This means that
scientists will be able to trace each health problem to a particular gene. It will then be possible to fix this
problem by fixing the responsible gene. The hope is that several addictions, such as smoking, alcoholism
and drug abuse, will be curable through gene therapy.
Which of the following assumptions is the above argument based on?
(a) Genetic manipulation is the sole way to cure addictions.
(b) In the future only genetics will matter in the world of medicine.
(c) Addictions are genetically induced, not caused by other factors.
(d) Every human being is addicted to something.
130. One morning after sunrise Deepak while going to school met Raj at road crossing. Raj’s shadow was
exactly to the right of Deepak. If they were face to face, which direction was Raj facing?
(a) South (b) North (c)East (d) West
Directions (Q.131): In each of the questions given below three statements are followed by some
conclusions. You have to take the given statements to be true even if they seem to be at variance from
commonly known facts. Read all the conclusions and then decide which of the given conclusions logically
follows from the given statements disregarding commonly known facts.
131. Statements:
Some songs are not tunes.
All tunes are drums.
Some tunes are sticks.
Conclusions:
I. Some drums being songs is a possibility.
II. All those drums which are tunes can be songs.
(a) if only I follows. (b) if only II follows. (c) if either I or II follows. (d) if both I and II follow.
Direction (Q.132-Q.134): In a family, there are six members- A, B, C, D, E and F. A and B are a married
couple, A being a male member. D is the only son of C, who is the brother of A. E is the sister of D. B is
the daughter-in-law of F, whose husband has died.
132. How F has related to A?
(a) Mother (b) Sister-in-law (c) Sister (d) Mother-in-law
133. How is E related to C?
(a) Daughter (b) Aunt (c) Cousin (d) Sister
134. Who is C to B?
(a) Brother (b) Brother-in-law (c) Son-in-law (d) Nephew
Direction(Q.135): In the question below are some conclusions are given followed by set of statements.
You have to decide from which set of statements the given conclusions logically follow(s) disregarding
commonly known facts.
135. Conclusions:
I. No puzzle is seating.
II. Some puzzles are coding.
III. Some syllogisms are not seating.
Statements:
I. All puzzles are coding. No seating is syllogism. Some syllogisms are puzzle.
II. No coding is seating. All syllogisms are puzzle. Some Puzzles are coding.
III. Some puzzles are syllogism. All syllogisms are coding. No coding is seating.
IV. No seating is puzzle. Some puzzles are syllogism. All syllogisms are coding.
(a) Only Statement I follows. (b) Only Statement II follows.
(c) Only Statement III follows. (d) Only Statement IV follows.
Head Office: 127, Zone II, MP Nagar, Bhopal |+91-7676564400| https://www.toprankers.com Page 33 of 36
6 months in India, here in this case facts are clear and still consider themselves the authority on these
that he is a UK resident, thereby provisions of this topics. Option b) is also correct as the author
visa regime won’t be applicable on him, and his visa mentions that monetarism's assumptions had
will be rejected. Thereby making option b & d become accepted as common sense until the 2008
incorrect. Option c is incorrect it merely states a fact recession. Option c) is correct as it is mentioned in
but does not give any reasoning. the first paragraph that economics is trying to solve
103. (c) Passage clearly states that India does not have a problems that do not exist. Therefore, option d) is the
refugee policy and grants shelter to foreigners facing answer. It is not an inference because it is already
persecution in their home country on a case to case mentioned in the last paragraph.
basis. Here in this case the passage whilst is about 108. (c) Option a) is incorrect as this was an assumption of
Afghan nationals fleeing from their country, it does monetarism in the 1980s and not something that the
state on case to case it can grant shelter, further as author deduces. Option b) is also incorrect as the
the passage is not talking about illegal immigrants last paragraph mentions that modern economists
hence, we cannot comment on it. Thereby making have failed in finding solutions to these problems.
option b and d incorrect.Option A is incorrect as it is Option d) is also incorrect as the author disapproves
against the facts as Sharbat Gula is Afghani and of this shift of economics towards mathematics.
Ashraf Ghani Bangladeshi. Option d is incorrect for Hence, option c) is the answer which is mentioned in
the passage talks about nothing in relation to right to the second paragraph of the passage. Refer to the
dignity, hence can be ruled out. lines, ‘This continues to be the case, despite the fact
104. (d) The pre requisite laid by the passage for a Afghan that, since the 2008 recession, central banks have
national seeking refuse in India is by applying for e- been printing money frantically in an attempt to
Emergency X-Misc Visa for 6 months, and no other. create inflation and compel the rich to do something
Here in this case Ram has forged document to enter useful with their money, and have been largely
in India which by no means is lawful within the unsuccessful in both endeavours.’
meaning of the passage, hence he will not be 109. (a) Refer to the lines, ‘The deliberate or unintentional
allowed to enter in India. Option b for it is making spread of misinformation, despite capturing
assumptions that done crimes in Afghanistan widespread public attention, remains as rampant as
including forging passport which are not provided for ever…’ and ‘Misinformation is a long-standing
in factual matrix. Option c is incorrect for it is not problem that demands long-term, sustainable
addressing the question of whether he will be solutions as well as short-term interventions.’ Option
allowed to enter India. Option A is incorrect for it is a (a) is true as the author states that misinformation
fact-based answer having no relevance to the has been existing for a long time.
passage. Option (b) may be true but cannot be ascertained
105. (c) Legal reasoning questions are always answered from the text of the passage. The author is strictly
basis the principle/ passage. Here in the passage,it against the spread of misinformation. Plus, morally it
is clearly provided if an Afghan national is provided is not advisable that a bitter truth of the society be
a visa its is for the purpose of facilitating their stay, hidden and a sugar-coated lie be told around. It
here as the facts irrespective of the fact that she is a actually worsens maters, especially during pandemic
Muslim she will be granted the stay for she has times.
procured the visa. Furthermore, CAA bars citizen, Option (c) is incorrect and can be sourced from the
Shyra has seeked refugee not applied for citizenship last paragraph of the passage. The author puts the
of the country. Thus, she will be allowed to stay, onus of fighting misinformation both on the
thereby making option b and d incorrect. Option a is institutions as well as the people. Refer to the lines,
incorrect for it is out of scope the passage, passage ‘It hasn't been enough. If we're to stop
does not discuss the concept of equality. misinformation and its insidious effects, we need to
radically expand and accelerate our counterattacks.
SECTION - D : LOGICAL REASONING It will take all sectors of society: business, non-
profits, advocacy organizations, philanthropists,
106. (b) The author through the second paragraph wants to researchers, governments, and more.’
communicate that the assumptions of the 110. (d) Option (a) won't affect the author's claims as the
economists and the actions of the governments author believes that misinformation is a menace that
since the 2008 crash still continue despite yielding the world is fighting. Thus, this option will strengthen
no success. According to the author, we should have the author's viewpoint.
learnt our lessons well and should have looked at Option (b) can't weaken the author's arguments
fresh alternatives for better economic understanding because the author has not mentioned in the
and sustenance. Option b) best reflects the thoughts passage as to who is spreading the most
of the author. misinformation. He is much more concerned about
Option a) is incorrect as the passage doesn't mention how to tackle this menace. Thus, this statement has,
anywhere that we cannot do anything to improve the in fact, no bearing on the author's argument.
situation. Option c) is also incorrect as it only covers Option (c) can't weaken the author's argument. It has
an example from the passage and not the main point. simply no relevance to the author's argument.
Option d) is incorrect as the author talks about much In light of the last paragraph of the passage, it can
more than teaching. Hence, option b} is the answer. be inferred that the author emphasizes that all and
107. (d) Option a) is correct as the passage lists the problems sundry should come forward to counterattack the
economists are supposed to solve but fail to do so problem. But if the major issue is organizations and
Head Office: 127, Zone II, MP Nagar, Bhopal |+91-7676564400| https://www.toprankers.com Page 9 of 12
the people are behind eradicating misinformation, option (c) is the correct answer. Option (a) is close,
then this assertion is false. Therefore, Option (d) is but does not take into consideration of the reason
the correct answer. behind its origin. Option (b) and (d) serve as
111. (c) Statement 1 is an instance of an unintentional introductions and are incomplete as the core
spreading of misinformation by an innocent person. messages.
While Steve spreads it deliberately, even though it is 115. (a) Refer to the third part of the passage. The passage
a prank, it is misinformation; Tony spreads it very clearly states “The founding philosophy of the
unknowingly. The author has mentioned this in the Web was for it to serve as a democratizer and
first line. Thus, statement 1 is mentioned. equalizer, to empower the long tail and eliminate
Statement 2 is an instance of a person leaving social monopolies and intermediaries. The Web did solve
media after getting frustrated from all the three big problems for us: an information problem
misinformation. Though the author mentions people with search and wikis, a communication problem
finding it difficult to believe everything on the internet, with email and messenger tools, and a distribution
he doesn't mention people quitting social media problem with file-sharing and e-commerce. But it
altogether. Thus, statement 2 is not mentioned. could not address the two big problems that it was
Statement 3 highlights how social media undertakes supposed to solve: one of trust and security, and
a short-term measure to fight the problem of another of disintermediation—its original
misinformation. The author has mentioned it in the philosophy.” Which make option (b) and (c)
passage. Thus, option (c) is the answer. incorrect. Option (d) is contrary to what is mentioned
112. (c) To arrive at the rationale of the answer, we need to in the passage. Refer to the line, ‘A lot of us think that
carefully read the question stem that demands the the World Wide Web and the internet are the same,
most significant agent. ‘Most’ means that others also but they aren’t.’ Corroborate means to confirm the
play their part but one that is most effective as an given information, which is not the case in the
agent. In expression like ‘most’, there will be more passage.
than one close options. One has to choose the 116. (a) We share our information with the technology
closest to what has been demanded as the answer. companies and they have control over those
Statement 1 is not correct as the people need to be information. So, option A is the most apt choice.
careful not to spread misinformation; the reason Rest of the sentence are not related to the passage.
given in the option is insufficient. Also, it is not the Thus, option (a) is the correct answer.
most significant as the process is slow, as mentioned 117. (b) The meaning of the following words:
in the passage. The reason is that a change in Dismal: filled with hopelessness.
people will help tackle the issue in a better way, but Hopeful: feeling or inspiring optimism about a future
in the long run. event.
Statement 2 is correct as the author points out that Pensive: thinking in a quiet way, often with a serious
the companies, though, not solely responsible expression on your face.
agents for this menace, have a better chance of Confrontational: challenging and argumentative.
curbing the menace as they have the technical and The passage ends with stating that blockchain
logistical aids. Instant blocking of miscreants technology is going to take back the autonomy which
spreading the information and a sound warning will web intended to when it was invented, that is a space
be a deterrent. which is free of intermediaries. The passage ends
Statement 3 is incorrect as the actions by with a solution at hand. The author leaves the
government will bring about a change, but in the long passage on a positive note. Thus, option (b) is the
run. Political fraternities have influential people. They correct answer.
can educate the people far better as not to spread 118. (c) The passage states that “: The NFT project was his
misinformation than anyone else, is certainly true but “first foray into crypto", but he saw similarities in his
they cannot be the most significant agents. original vision for the web and the philosophy behind
Thus, option (c) is correct. the decentralized network of Ethereum’s blockchain,
113. (b) The author in the last paragraph clearly provides that which underpins most NFTs.” Thus, option C is the
the solutions suggested and implemented before is correct answer. Option (a) is incorrect as the phrase
not enough. Further, the battle cannot be won by ‘complete imitation’ finds no mention in the passage.
only investing in the supply side of the information. Option (b) does not find support in the passage.
Therefore, the correct answer is Option (b). Since, Option (d) is contrary to the passage. Read the quote
the main statement is explicitly mentioned in the above.
passage, it cannot be probably true; therefore, option 119. (c) (a}- It can't be derived from the passage as there is
(a) is incorrect. The main statement does not state no evidence implied or explicitly mentioned which
anything that defies the content given in the points towards his belief in God. {b}- It is out of scope
passage, makes option (c) and (d) incorrect. as the context in which the argument was made has
114. (c) The passage talks about how web had certain vision, no relation with biological activity at micro level. (d}-
as the passage states “serve as a democratizer and This would undermine the premise made by Laplace
equalizer, to empower the long tail and eliminate as it would open doors for the possibility of existence
monopolies and intermediaries.” However it got of a creator. (c}- It is correct as the premise is
deviated and finally the block chain technology might possibly based on the fact that Laplace might have
bring “closer to the original vision of Time Berners- proved or formulated the existence of all events in
Lee and his co-conspirators.” Thus, option C tile Universe on the basis of physics and
encompasses the theme of the passage. Thus,
Head Office: 127, Zone II, MP Nagar, Bhopal |+91-7676564400| https://www.toprankers.com Page 10 of 12
mathematical equations. Hence, (c} is the right 125. (d) The passage clearly states how putting the shoes on
answer. covers us with opinions and dogmas and then when
120. (b) (a)- It is irrelevant in the context of the question as it we see things, they get shaped by our vision. Our
talks about the definition of ions. (c)- It talks about a vision of anything isn’t a true picture and that the
change brought about by suspending the apple from source of our happiness isn’t the reality but how the
string and explaining its motion. This doesn't shoes presented things in front of us. All of the
undermine the argument asked in the question. {b}- following reflect the psychological shoe.
It talks about not accurately determining all the 126. (b) To have an objective clarity about happiness,
variables at any given point of time, which certainly unhappiness and anything in life, the opinionated
undermines the arguments of Knowing initial values shoe needs to be left out. The opinionated shoe here
of all the variables involved. Hence, {b) is the right is the psychological shoe we put on, and such a shoe
answer. Since option (b) is the correct answer, option prevents us from having a purer connect with reality,
(d) is ruled out. life, things, and happiness in general. When we have
121. (b) (a), (c} and {d)- they are additional information which that shoe on, our views are fared by the opinion we
are essentially out of the scope of the given passage hold of them. For example- someone who doesn’t
and therefore, can't be said to be inferred. Option (a), like the colour red will shape his view of red rose on
by stating different outcomes suggests a contrary it (with shoe on) but that’s a personally inclined
view to Laplace. Option (c) is too far-fetched to be choice as per the author. Option (a) is vague. The
inferred. It must take into account various intertwined phrase ‘wrong prism’ does not provide clarity. Option
factors before inferring something that abstract. (c) is close, but not the MOST accurate. Option (d) is
Option (d) is contrary as an inference. {b)- it can be far-fetched.
inferred from the last few sentences of the passage 127. (c) Refer to the lines, ‘If you analyse it, you will see that
which provide the example of human body (made up if we are unhappy in life, it is not because of what is;
of small particles electrons, protons and neutrons} it is because of our opinion of how life should be.
and how it operates. Hence, {b} is the right answer. When our view of life conflicts with our life, this
122. (d) The question stem attempts at weakening the makes us unhappy.’ The correct option is (c)-our
content of the passage by introducing a question that psychological shoe, that prevents us from
challenges the essence of the philosophy that if you connecting and analysing things in their natural
know the initial values of the variables that colour and not through gauging the true meaning.
characterise a physical system, together with the Option C is the gist of the passage which also
equations that explain how these variables change supports the question stem. In option (a), the author
over time, then you can calculate the state of the does not use ‘only reason’ as the answer. Option (b)
system at all later times. It means that a latter event is contrary to the passage as it is a conflict and not
is not a deviation from the trajectory. It is expected consonance (harmony) in our perception and reality
and explained for. that is the root-cause of unhappiness. Option (d) is
All the options given here are either agreeing or are out of scope.
far-fetched and way outside the scope of the 128. (a) conclusion statement. The main argument is that we
passage. Option (a) answers the facts given in the never see a flower as it is. The statement given can
passage and is not aligned with the question stem. be logically deduced as our vision is shaped by our
Option (b) is more of ‘thinking out loud’ based on the opinion, and the psychology of our likes and dislikes.
content of the passage. Option (c) corroborates with 129. (c) As we are now aware, an assumption can be tested
the content of the passage. So, no option is likely to by checking if the argument still holds if the opposite
be true in case of the situation mentioned in the of the assumption is assumed to be true. If the
question as options are unrelated to the situation opposite of (C) is true, then addictions are caused
given. Hence, (d) is the right answer. due to non-genetic reasons, meaning that the
123. (d) (a)- It is out of scope of the passage as it talks about applications of mapping the human genome cannot
free-will sceptics, which isn't discussed in the cure addictions such as smoking, alcoholism and
passage and can't be the main theme of the drug abuse. Only negating choice (C) invalidates the
passage. Also, it is contrary to the passage. {b}- It is conclusion. Hence, C
incorrect as per the last sentence of the second 130. (b) North. In morning the shadow falls towards the west.
paragraph of the passage. (c)- There is no evidence Raj’s shadow falls to the right of the Deepak. So Raj
or example quoted in the passage which points is facing North.
towards this argument; in fact, all the events quoted 131. (d) if both I and II follow.
has an explanation in Physics and is well The basic diagram for the given statements is:
determined, which is mentioned in option {d} and is
the gist of the passage. Hence, (d} is the right
answer.
124. (b) Using the power of the mind to think freely. The mind
is entitled to think and shape views and opinions
based on its power of free thinking. Not thinking with
pre-determined conditionings. By calling it the grand
prerogative, it is highlighted that such a privilege
exists with humans and great ideas, opinions etc
come from it. Rest of the options do not reflect the
correct interpretation of the phrase. The possible diagram for both conclusions I and II
is:
Head Office: 127, Zone II, MP Nagar, Bhopal |+91-7676564400| https://www.toprankers.com Page 11 of 12
Hence, both I and Ii follow.
Hint (Q.132-Q.134) We have finally reached in the
position of completing our tree diagram by Total = 300
incorporating the final information which is about F Bays : Girls = 3 : 2
who is a female since her husband had died. And B Boys = 180, Girls = 120
is her daughter-in-law. Therefore, she’s the mother 141. (d) Girls who passed only in Legal paper: 50
of A and C. 142. (a) Reqd % = 63 × 100 /300 = 21 %
132. (a) is the correct answer. 143. (d) Total students who passed in Legal = 45 + 50 + 40 +
133. (a) is the correct answer. 72 = 207
134. (b) is the correct answer. 144. (c) ratio = 72 + 63 /50 = 135/50 = 27/10 = 27:10
135. (d) is the correct answer. 145. (d) Students who passed at most in one subject = 45 +
I. True II. True III. True 50 + 30 + 63 = 188
Explanation (Q.146-Q.150):
146. (d) Number of passengers in train S = 24% of 10000
Number of passengers in train M = 20% of 10000
Number of passengers in train L = 15% of 10000
(24%+20%+15%)𝑜𝑓 10000
∴ Required average =
3
59% 𝑜𝑓 10000
= = 1966.66 = 1966
3
147. (a) Number of Passenger’s in train
9
R = 10000× = 900
100
100−34−26 900×40
SECTION - E : QUANTITATIVE TECHNIQUES ∴ number of males = 900 × = = 360
100 100
148. (d) Percentage of people in train Q=19%
Explanation (Q.136-Q.140): Percentage of people in trains A and R together
Number of Savings Accounts = (13+ 9)% =22%
19
= 24 × 2050 /100 = 492 ∴ Required percentage = ( × 100) % = 86%
22
Number of D Type = 2050 × 1 /5 = 410 Note: As we have already discussed, we don’t need
Number of C Types = 16 × 2050 /100 = 328 to take actual values here. We can find the required
Number of E Type and B Types = 820. percentage by taking only their percent terms also.
Number of B Types = No. of E Type + 182 149. (d) Train M has second highest number of passengers
E Type + B Types = 820 as it has second highest percentage.
or E Type + E Type + 182 = 820 150. (d) Percentage of passengers in train M =20%
or, 2 E Type = 820 - 182 = 638 Percentage of Passengers in train L = 15%
E Type = 638/2 = 319 20−15
B Type = 319 + 182 = 501 ∴ Required percentage =( × 100) %
15
136. (b) Reqd ratio = 410/820 = 1:2 𝟓𝟎𝟎
= % ≈ 𝟑𝟑%
𝟏𝟓
137. (c) Number of non- oprative accounts = 410 × 20 / 100
= 82
Number of accounts which are operative = 410 – 82
= 328
138. (d) Reqd% = 328 × 100 /902 = 36.36 ≈ 36 %
139. (c) Total number of E Type, C Type
and D Type = 328 + 319 + 410 = 1057
140. (a) Difference = 319 + 492 – 501 = 811 - 501 = 310
Explanation (Q.141-Q.145):
Head Office: 127, Zone II, MP Nagar, Bhopal |+91-7676564400| https://www.toprankers.com Page 12 of 12
SECTION - D: LOGICAL REASONING
Direction (Q.106-Q.110): Read the passage given below and answer the question that follows:
The latest World Inequality Report makes for a sobering read. It details the rising levels of income and wealth
inequality across countries. As far as income is concerned, the richest 10 percent of the global population
currently takes 52 per cent of global income, whereas the poorest half of the population earns 8.5 percent of it.
The picture is worse when it comes to wealth inequalities.
India is one of the worst performers. “India stands out as a poor and very unequal country, with an affluent elite,”
states the report. While the top 10 per cent and top 1 per cent hold respectively 57 per cent and 22 per cent of
total national income, the bottom 50 per cent share has gone down to 13 per cent. It is not just the inequality in
income and wealth that plagues India. The report also points to extreme gender and carbon inequality. For
instance, at 18 per cent the female labour income share in India is one of the lowest in the world. Inequality is
so high in the country that when India is removed from calculations, the global bottom 50 percent income share
rises.
Alarming as these findings are, they are not entirely surprising. That’s because in India’s case the primary
instrument — that is, fast economic growth — to reduce poverty and counter inequality has been faltering for a
while. GDP growth has been rather iffy since the Global Financial Crisis of 2008 and has completely lost its
momentum since the start of 2017. For a relatively poor country such as India, the most durable and dependable
way to reduce inequality is to increase the size of the GDP. That is the first policy lesson for the government.
However, as evidence from across the world has shown, fast GDP growth alone doesn’t help, especially when it
comes to tackling inequalities in accessing education and health. That is the second key policy lesson. A good
starting point in this regard would be for the government to improve the quality of data on inequality within the
country.
106. Which of the following reflects the main idea of the passage?
(a) It is rather perplexing that India is one of the worst performers according to the World Inequality Report.
(b) There are a few key lessons that India needs to learn from the latest World Inequality Report.
(c) India could have easily shown well in the latest World Inequality Report, had it learnt the lessons from the
previous report.
(d) The primary objective of the World Inequality Report is to draw lines between income and wealth of an
individual.
107. What is the role of the first paragraph in the given passage?
(a) The first paragraph contains the main idea of the given passage.
(b) The first paragraph gives reasons on why India is the worst performing country.
(c) The author briefs the readers about the passage in the first paragraph.
(d) The first paragraph plays an insignificant part in the passage.
108. What is the reaction of the author on India’s dismal performance in the World Inequality Report?
(a) The author was perplexed at India’s performance and blames it on the administration.
(b) The author expected India to perform even worse in the given report.
(c) The author was not completely surprised and presented a few lessons regarding it.
(d) The author is not surprised at all on India’s dismal performance.
109. Which of the following is not one of the findings from the World Inequality Report?
(a) There is no difference between the income levels and the wealth levels.
(b) A minority of the population holds the majority of wealth.
(c) In India, the top ten percent holds close to sixty percent of the national income.
(d) India presents a dismal picture when it comes to wealth inequalities.
Head Office: 127, Zone II, MP Nagar, Bhopal |+91-7676564400| https://www.toprankers.com Page 25 of 36
110. According to the author, which of the following steps may help India improve its performance in the next
inequality report?
I. By only improving GDP growth.
II. Increasing the size of GDP.
III. Improving the quantity of data on the country’s inequality.
(a) Only II (b) Only I and III (c) Only II and III (d) All I, II and III
Direction (Q.111-Q.115): Read the passage given below and answer the question that follows:
Happy families are all alike; but every unhappy family is unhappy in its own way. This is what Leo Tolstoy
thought in his time. But the twenty-first century seems to have turned Tolstoy’s argument on its head. Unhappy
families now seem to have one — singular — thread of discontent: difference of opinion. Hard data may be
difficult to come by just yet, but psychologists seem to be echoing news reports that suggest that more and more
young people are choosing to cut ties with parents over clashes in cultural values. Formally known as
‘estrangement’, most of these ‘break-ups’ between parents and children tend to be initiated by the latter. Families
have always harboured differences of opinion: loud arguments are the hallmark of, say, the argumentative Indian
clan. The disapproval of individualism and juvenile autonomy — purportedly another leading cause of
estrangement — within familial settings is not new either.
What has changed, according to experts, is a growing awareness of mental health, leading, in turn, to deeper
appreciation of how toxic or abusive family relationships can affect psychological well-being. There has also
been a corresponding rise in self-dependence; fewer children are now reliant on their parents for livelihood or
property. However, the principal cause of estrangement — rifts based on political opinion and values — is a
growing area of concern. In the course of its evolution, the family has found ingenious ways of fusing the political
with the personal. What, then, has changed so fundamentally? The answer seems to be the prevailing —
deepening — crisis in consensus. Individuality, it would seem, has birthed an inflexibility of opinion, so much
so that a difference in views is now tantamount to a moral transgression. The fractured political constituency is
encouraging these chasms within other institutions. Social media — that other fount of shrill opinion and false
narratives — is equally dismissive of divergent points of view.
What an embittered, opinionated but divided family — and the world? — needs are greater investments in
conversation, empathy and consensus. Isolation — a social pathology that is spreading like a pathogen — breeds
ignorance. The challenge across social settings is to debate differences without demonizing them.
111. Which among the following best represents the purpose of the author?
(a) The author apparently does not have any particular purpose for writing this passage as many conflicting
opinions have been expressed.
(b) The author tries to find out why the entire world is moving on one single path despite the existence of multiple
paths.
(c) The author is exploring the reasons for the new relationship that has surfaced between young adults and their
families.
(d) The author focuses on exploring the reasons for increasing fractured families due to difference in political
ideologies, among other factors.
112. Out of the following, which one is most likely to be an opinion held by the author?
(a) There may be some recourse to a divided family - they can still be united.
(b) An Indian family is not likely to be divided at all - they have always harboured differing opinions.
(c) Juvenile autonomy is one such cause of more families getting divided which was not present earlier.
(d) Unhappy families now seem to have one — singular — thread of discontent: difference of cultural set up.
Head Office: 127, Zone II, MP Nagar, Bhopal |+91-7676564400| https://www.toprankers.com Page 26 of 36
113. Out of the following, which can weaken the author’s arguments?
(a) Individuality has begotten rigidity of opinions; and the differences in opinion are seen as moral offence.
(b) It is extremely difficult to find out a precise reason for families getting divided.
(c) One of the reasons for the estranged family is the awareness of mental health and children want to dissociate
from parents who are toxic to their mental health.
(d) One can be embittered over a difference of opinion, but division of family is not the solution.
114. Which of the following can be inferred from the given passage?
I. The author of the passage is in agreement with the arguments presented by Leo Tolstoy.
II. If members in the families do not perceive differences of opinion as moral transgression, it is likely that there
would be fewer families getting divided.
III. Reducing the political differences within the families would stop families from estrangement.
(a) Only I (b) Only II (c) Both II and III (d) All I, II and III
115. According to the second paragraph of the passage, which of the following has not led to estranged families?
(a) Differences in opinion among the families leading to rifts between them.
(b) Growing awareness of mental health leading to realization of how good families can be to one’s well-being.
(c) More young people getting self-dependent, relying less now on their parents for livelihood or property.
(d) Social media — that other fount of shrill opinion and false narratives — is equally dismissive of divergent
points of view.
Direction (Q.116-Q.120): Read the passage given below and answer the question that follows:
Alongside overseeing land administration, revenue collection, health, education, disaster management and other
goings-on in the district, Madhya Pradesh collectors will, henceforth, judge ‘entertainment’ as well. Recently,
the MP home minister announced that the state government has finalised guidelines whereby anybody wanting
to shoot a film, OTT drama or even an advertisement will have to submit a complete script to the collector’s
office, and the officer will decide if there is anything ‘objectionable’ in it before saying yes or no. Presumably,
MP, a relatively poor state that can do with all the business it gets, doesn’t mind losing the film shoot business.
It’s quite likely that filmmakers, blessed with an abundance of choices in India and abroad, will sidestep a state
where local administrators will be culture arbiters.
The larger point is of course that the MP's decision is terrible news in a democracy, and one that prides itself in
the soft power of its film and entertainment industry. India is experiencing some kind of a filmmaking paradigm
shift as OTT productions, as well as some movie hall releases, made here get good response from critics and
audiences abroad, and movies made in dozens of countries around the world are seen and liked by Indians. India
is a major market and content producer for OTT because it has the audience and the talent – but this rich
ecosystem is predicated on creative freedom.
MP has pioneered a rule that, if replicated, can easily deal a body blow to that freedom. Bad ideas have a way of
replicating across states – witness how many states have adopted the self-defeating rule on private industry
reserving jobs for locals. If other state governments want to prove they are as “culturally conservative” as MP,
a flourishing industry with a global footprint will suffer. So will India’s reputation. And that has consequences
outside the film industry.
116. Which of the following best sums up the main idea of the passage?
(a) A blow to India’s reputation would have consequences outside the film industry.
(b) The recent announcement in Madhya Pradesh of local administration as culture arbiters and scrutinizers is
against the creative freedom and business of the film industry.
(c) The recent announcement in Madhya Pradesh of local administration as culture arbiters and scrutinizers is a
harbinger of doom to India’s democracy.
(d) The film industry needs to regulate itself, else the government would step in.
Head Office: 127, Zone II, MP Nagar, Bhopal |+91-7676564400| https://www.toprankers.com Page 27 of 36
117. Out of the following, which can be inferred from the passage?
(a) The broader consequences of the development in MP would be elsewhere, other than the blow to the film
business.
(b) The Indian film industry is already on a downward curve, with the new development accelerating the decline.
(c) There is an increasing behavioural change of Indian filmmakers making films outside the country.
(d) The officer would likely give the nod to the script, which he finds ‘objectionable’.
118. For which of the following claims has the author not presented any evidence or information to strengthen?
I. Madhya Pradesh’s decision is bad news for democracy.
II. Indian films are doing average in other countries.
III. Madhya Pradesh collectors have other departments to take care of than the entertainment industry.
(a) Only I (b) Only II (c) Only III (d) I, II & III.
119. The author argues that creativity may suffer due to MP’s new rule. What among the following, if true, can counter
this argument?
(a) The filmmakers would leave the film industry and look for better opportunities.
(b) The new rule in MP is limited to filmmakers taking permission for shooting in sensitive areas.
(c) The filmmakers would make the content such that their scripts get a nod from the relevant officer.
(d) The filmmakers will file a petition in the Supreme Court against the decision.
Direction (Q.121-Q.125): Read the passage given below and answer the question that follows:
After years of stability, the Office for National Statistics reported that alcohol-specific deaths in 2020 had
increased by 19% from 2019, counting 8,974 deaths coded as caused by alcohol misuse, with three in four being
from alcoholic liver disease. The national lockdown saw an increase in abstention, but also in heavy drinking.
Harm can increase without greater total consumption: the distribution matters, not just the average.
Drinking too much alcohol raises the risks of many diseases, so analysts try to estimate total numbers of deaths
attributable to alcohol consumption – they modelled about 19,200 alcohol-related deaths in England in 2019,
around four times the direct count of number of alcohol-specific registrations. The method requires many
assumptions, such as 11% of breast cancer deaths in women over 75 being due to alcohol, and a major recent
change in such attributable fractions cut estimated alcohol-related death figures by around 23%.
People might like a single number to settle an issue but we cannot count everything directly and analysts may
need to construct a range of statistics to improve our understanding. Many tens of thousands of deaths every year
are attributed to air pollution, but until an inquest in 2020, it was never given as an official cause of an individual
death.
Influenza also demonstrates the limitations of counting. Between 2013 and 2020, only around 600 people in
England and Wales died with influenza as the direct underlying cause each year. Yet England’s public health
agency estimated in 2020 that there were around 15,000 “influenza-attributable deaths” in the 2016-17 season
alone. That figure comes from the FluMomo model, which picks out periods with high mortality over a curved
seasonal baseline, attributing those deaths to influenza or extreme temperatures. This approach has some
weaknesses, including potential overestimation and insufficient corrections for registration delays.
Head Office: 127, Zone II, MP Nagar, Bhopal |+91-7676564400| https://www.toprankers.com Page 28 of 36
121. Which of the following is the main message of the author in the passage?
(a) Deaths attributable to alcohol, air pollution and flu must be modelled rather than counted.
(b) Explaining a health issue is close to impossible, with so many factors that need to be taken into account.
(c) Analysis of various factors is really crucial, before making any deduction about a health issue.
(d) A health issue that impacts the entire world is much more difficult to explain than the one whose impact is
endemic.
122. Which of the following can be definitely deduced from the passage?
(a) 11% of women over the age 75 who died due to breast cancer died due to alcohol.
(b) In the year 2016-2017, England saw over 15000 people die due to influenza.
(c) There were a total of 8794 alcohol related deaths globally.
(d) None of the above
125. What role does the following statement (based on the last information given in the second paragraph) play
towards the arguments of the author, “An inquest aids in attributing some health issue to an official cause of an
individual death.”?
(a) This statement is an assumption made by the author.
(b) This statement can be labelled as one of the supporting ideas of the passage.
(c) This statement is a claim made in the passage without any supporting evidence.
(d) This statement plays no part in the passage.
Direction (Q.126-Q.130): Read the passage given below and answer the question that follows:
How should schools reopen after the pandemic closure? In the midst of discussions about learning deficits,
remediation, accelerated learning, and so forth, what is clear is that it should not be business as usual. Education
is not a race. It is a child’s journey to fulfilling his potential. Reopening schools should be an opportunity to
rethink the teaching and learning process itself.
Tagore became one of India’s foremost philosophers of education. As a child, he had dropped out of formal
school and was taught at home. For him, education meant much more than rote learning: “The highest education
is that which does not merely give us information but makes our life in harmony with all existence.” He dismissed
any pedagogy that sought to cut children off from the world around them: “We are made to lose our world to
find a bagful of information instead. One of the students at this school in rural Bengal was Amartya Sen. In his
memoir Home in the World (named after Tagore’s classic novel of the freedom movement, Ghare Baire or
‘Home and the World’), Sen writes about his years at Santiniketan. In the Dhaka school where he began his
education, he had not at all been motivated to study. Contrary to the principal’s expectations that all his students
should “shine”, he had ranked 33rd in a class of 37.
Head Office: 127, Zone II, MP Nagar, Bhopal |+91-7676564400| https://www.toprankers.com Page 29 of 36
In 1941, due to fears of the possible Japanese bombing of the cities, Amartya was sent to Santiniketan. Here, he
discovered the freedom of learning at his own pace. There was no pressure to excel in terms of grades or exam
performance. This was immediately liberating: “I became what would count as a good student only when no one
cared whether I was a good student or not.” Indeed, there seemed to be an inverse relationship between grades
and originality.
Santiniketan’s approach was to help the whole child to learn through exploration - art, music, curiosity, and the
careful observation of nature. The environment was stimulating. There was no corporal punishment. “School
was fun in a way I had never imagined school could be,” writes Sen. The atmosphere of freedom and thoughtful
reasoning shaped his educational attitudes, including his response to inequities around him: For example, by
running night classes for tribal children from neighbouring villages.
There are many lessons here for the Indian education system today. Joyful and creative learning should not be
an indulgence meant only for privileged children. Every child should be able to learn in an atmosphere that is
free, reflective and affirming. They should be able to relate new concepts to what they are already familiar with
within their own lives.
Karnataka’s Vidyagama programme, which began with a group of committed teachers creating informal, outdoor
learning circles or “vataara shaale” during the pandemic-where children gathered with a teacher for in-person
teaching in small groups, in outdoor community spaces; not bound by blackboards and textbooks, but learning
interactively, through stories and activities - has been acknowledged as an example of an alternate model of
teaching and learning.
The pandemic itself can be a starting point for inquiry-based learning: Children can be encouraged, within the
safe space of the learning circles, to discuss what they saw, experienced and learned during the pandemic.
Rather than being cooped up inside small, cramped schoolrooms through the day, extending lessons into outdoor
spaces, where feasible, will also improve ventilation, which is perhaps next in importance only to vaccination
during the continuing pandemic.
Symbolically, too, an open classroom presents itself to fresh winds and ideas. We should really open up our
education system.
126. What is the main idea of the passage the author is stressing over?
(a) To start giving education in open premises rather than being cooped up in small classrooms to improve
ventilation and present itself to fresh winds and ideas.
(b) To take pandemic as an opportunity to re-think the education system because education meant much more
than rote learning and is not bound by blackboards and textbooks.
(c) To fill the gaps present in the Indian Education system and to improvise it as per the industry set-up.
(d) To understand the need for improving the education system.
Head Office: 127, Zone II, MP Nagar, Bhopal |+91-7676564400| https://www.toprankers.com Page 30 of 36
128. Why is the author stressing over the topic “post-pandemic opportunity to re-think education”?
(a) Now everything is going online and so does the education system.
(b) Post- pandemic, the education and examination system are completely disturbed now.
(c) Reopening schools should be an opportunity to rethink the teaching and learning process itself.
(d) Post pandemic, the schools should completely overhaul the education system as it is redundant.
129. What is/are the lesson/ (s) the author talked about in the passage for the betterment of the Indian education
system?
(a) Learning interactively, through stories and activities
(b) Every child should be able to learn in an atmosphere that is free, reflective and affirming.
(c) Both (a) and (b)
(d) Children figure out the smart approach to learning despite the restrictive information.
Direction (Q.131-Q.135): Read the passage given below and answer the question that follows:
The government’s new definition of a surrogate mother is a woman genetically related to the intending couple,
married with a child of her own, aged between 25 years and 35 years allowed to be a surrogate only once in her
lifetime. The intending couple meanwhile requires an official certificate to opt for surrogacy. On August 5, the
Lok Sabha passed, with almost no debate, The Surrogacy (Regulation) Bill, 2019. The surrogacy bill prevents
single parents, same-sex couples, divorced or widowed persons, transgender persons, live-in partners, and
foreign nationals from using a surrogate mother. The bill now awaits assent by the Rajya Sabha, the Parliament’s
upper house.
I. Proven infertility through diagnosed medical conditions should indeed be the only criteria to permit
surrogacy, said Manasi Mishra from the Centre for Social Research.
II. The bill says that the Centre and the State governments will appoint an appropriate authority to issue a
“certificate of essentiality” and “eligibility certificate” to the intending couple.
III. If the bill becomes law, same-sex couples, single-parents, and live-in couples will not be allowed to opt for
surrogacy, as they currently can.
IV. "By denying surrogacy to these groups – single parents, homosexuals, transgenders – you are denying
them their rights, making it a regressive or limited view of what a family is”, said Gargi Mishra, a gender
rights lawyer at Sama.
V. Experts and activists said banning commercial surrogacy will also take away the livelihoods of women
who rented out their wombs and would deny women rights over their own bodies.
VI. Some said the ban should be followed by modification of adoption processes, so single people and
homosexual couples can have children.
131. Which of the given statement states a course(s) of action to be taken by the government to enforce the bill?
(a) Only V (b) Only II (c) Both I & IV (d) Only I, III & IV
132. Which of the given statement(s) criticises the actions taken by the government?
(a) Only VI (b) Only II (c) Both I & VI (d) Only IV & V
Head Office: 127, Zone II, MP Nagar, Bhopal |+91-7676564400| https://www.toprankers.com Page 31 of 36
133. Which of the following statement(s) support (s) modifications to the Surrogacy Bill?
(a) Only II (b) Only V (c) Both I & VI (d) Only II, III & V
134. Which of the following supports the Surrogacy Bill mentioned in the stanza?
(a) Only VI (b) Only I (c) Both II & IV (d) Only II, III & V
135. Which of the given statement(s) describes direct impact(s) of the Surrogacy Bill?
(a) Only VI (b) Only II (c) Both I & VI (d) Both III & V
Head Office: 127, Zone II, MP Nagar, Bhopal |+91-7676564400| https://www.toprankers.com Page 32 of 36
from this doctrine.” And “he proposes that the income, but those data are referral points to the
central philosophy that governs the proportionality author presenting the main point that india needs to
doctrine is to shield the individual and his/her rights learn a few key lessons for its dismal performance
from the arbitrary brute power of the state, and in the latest world inequality report. therefore option
conceives of the proportionality doctrine as having (b) is the best suited answer. option (a) is incorrect,
four components- namely- proper purpose, rationale as the author does not find india’s performance as
connection, necessity and balancing.” As both very surprising; therefore, the expression ‘rather
assertion and reasoning is correct thus it straight perplexing’ contradicts the argument in the passage;
away eliminate all the other options, thus options B, hence, cannot be the main point. option (c) is
C and D stands incorrect. incorrect since the author has not talked about the
104. (b) because as per the 3rd line of the passage state may previous inequality report in the passage. this may
make special provisions for the benefit of women have been inferred from the passage, but certainly
under Article 15(3) reserving seats in legislative and cannot be considered the main idea of the passage.
judiciary bodies is also one of such even of option (d) is incorrect as well. the primary objective
benefiting women. Option A is not correct as of the inequality report is not the primary objective
Reservations or quotas for women in Parliament and of the entire passage.
judiciary bodies, serve to correct a historical wrong, 107. (c) the correct answer is c. the first paragraph contains
caused by the structural inequality between the the data illustrated in the world inequality report. the
sexes for many generations, can be justified by author only briefs his readers to the facts as a
recourse to Article 15(3) because the differential support to the main arguments ahead in the passage.
benefits/burdens are aimed at mitigating the effects he talks about the world inequality report and some
of a concrete, historical and institutional inequality of its findings. hence, this option is correct. option
therefore the reservation for women is not violative (a) is incorrect since there is no evidence that
of article 14. Option C is not correct as in the judicial suggests the first paragraph contains the main idea.
bodies too, inclusion women’s standpoint and their option (b) is incorrect, as it does not talk about
insights are also, must to impart absolute justice to india’s performance at all. option (d) is also
all. Option B is the most appropriate choice and incorrect. if it were insignificant, it would not have
therefore correct. As (b) thus it automatically been included at all.
eliminate option D. 108. (c) the correct answer is c. as is clear from the last
105. (d) As the author in its passage has tried to highlight the paragraph, ‘alarming as these findings are, they are
fact that how the laws made for the welfare and not entirely surprising.’ also, he did present some
benefits of women, promotes the gender based key lessons regarding the performance. therefore, it
assumption thus are discriminatory toward women is correct. option (a) is incorrect, as ‘perplexed’
itself also as given in the passage too “These means puzzled. the author in a way expected the
stereotypes, premised on assumptions about socially results based on the india’s performance since 2017.
ascribed roles of gender which discriminate against option (b) is irrelevant and far-fetched. option (d) is
women, are against the spirit of Article 14 of the an extreme option. the last paragraph mentions that
Constitution” The basis of the adultery laws of he was not entirely surprised. this option claims that
Sugar Rush were precisely the kind of stereotypical the author was completely unsurprised.
gender-based assumptions that the author intended 109. (a) option (a) is correct. in the first paragraph, the author
to do away with: i.e., that women are passive mentions the income levels and the wealth levels
partners, lacking in sexual autonomy thus could not both. clearly, there is a difference between the two.
give consent on her own or her consent does not options (b), (c) and (d) have been mentioned in the
mean anything before the law therefore option A first and second paragraphs respectively. therefore,
and C could not be correct as author does not they are not incorrect.
appreciates such laws. Option B also incorrect as the 110. (a) statement ii is clearly mentioned in the last paragraph
author main concern in the passage is to highlight of the passage. refer to the lines, ‘for a relatively
how laws for welfare promotes gender based poor country such as india, the most durable and
assumptions and are discriminatory in nature dependable way to reduce inequality is to increase
therefore option D is the most appropriate choice. the size of the gdp. that is the first policy lesson for
the government. however, as evidence from across
SECTION - D : LOGICAL REASONING the world has shown, fast gdp growth alone doesn’t
help, especially when it comes to tackling
106. (b) the correct answer is b. the main idea of the passage inequalities in accessing education and health. that
is what the author wants to communicate through is the second key policy lesson. a good starting point
the passage. it is either communicated in the in this regard would be for the government to
beginning or towards the end. the beginning of the improve the quality of data on inequality within the
passage mentions the report and the dismal country.’ statement i is incorrect, as the passage
performance of india with regard to inequality in mentions that fast gdp alone doesn’t help. however,
Head Office: 127, Zone II, MP Nagar, Bhopal |+91-7676564400| https://www.toprankers.com Page 11 of 16
the author talks about improving the quality of data 114. (b) statement i cannot be inferred. refer to the lines,
on the country’s inequality, not improving the ‘happy families are all alike; but every unhappy
quantity, making statement iii incorrect. therefore, family is unhappy in its own way. this is what leo
the correct answer is a. tolstoy thought in his time. but the twenty-first
111. (d) the correct answer is d. the new trend is the fact that century seems to have turned tolstoy’s argument on
families are getting divided over differences in its head.’ this indicates that the author did not
social and cultural values, but more on differences acknowledge leo tolstoy’s philosophy as valid in the
in political ideologies. option (a) is clearly incorrect, twenty-first century. statement ii can be inferred
as the author talks on a topic of estrangement since the author labels differences in opinion as to
between children and parents; therefore, the author be the main cause of family estrangements.
has a purpose of bringing to light the social malaise therefore, it would not be far-fetched to infer that if
now more a trend in the indian families. option (b) the members of the family did not interpret every
is incorrect, as there is no clarity over what that difference of opinion as morally offensive, they
particular path the author emphasizes upon. option would not be estranged. statement iii, on the other
(c) is incorrect, as which new relationship between hand, is an extreme option. though the author
young adults and their families has come up, is not mentions that differences in political differences is
clear either. the principal cause, he mentions some other causes
112. (a) option (a) is the opinion held by the author. notice as well. therefore, the statement cannot be inferred.
that the author has presented some solutions to the the answer is option (B).
challenge at the start of the third paragraph. refer to 115. (b) the correct answer is b. the second paragraph
the lines, ‘what an embittered, opinionated but mentions that the growing awareness of mental
divided family — and the world? — needs are health has led to realization of how bad can toxic
greater investments in conversation, empathy and relationships be to one’s well-being. this is in
consensus. isolation — a social pathology that is contrast to what this option suggests. on the other
spreading like a pathogen — breeds ignorance. the hand, options (a) and (c) have been clearly
challenge across social settings is to debate mentioned in the second paragraph. refer to the
differences without demonizing them.’ therefore, lines, ‘there has also been a corresponding rise in
the author agrees that there may be some recourse to self-dependence; fewer children are now reliant on
a divided family. option (b) is an extreme option. it their parents for livelihood or property. however, the
cannot be said according to the author, an indian principal cause of estrangement — rifts based on
family is not likely to get separated just because it political opinion and values — is a growing area of
has always allowed differing opinions. option (c) is concern.’ option (d) indirectly fuels the discord in
incorrect as well. refer to the lines, ‘the disapproval the family; therefore is incorrect.
of individualism and juvenile autonomy — 116. (b) the correct answer is B. the author clearly indicates
purportedly another leading cause of estrangement that the development in mp, where local
— within familial settings is not new either. option administration will act as culture arbiters and
(d) is incorrect. the singular reason for families to be scrutinize every shoot before giving permission,
unhappy today is difference of opinion and not would likely cause the film industry business and
cultural set-up. freedom of expression to suffer. he also indicates
113. (b) option (b) is correct because if it is very difficult to that it would have an adverse impact on the
find out a precise answer as to why families are creativity of the film makers. option (a) is far-
getting divided, then the author’s argument that fetched, as an announcement in mp will not be a
families are getting divided due to differences in blow to india. it is an extreme case. option (C) is too
opinions and values would be weakened. options extreme to be the main ideas (look out for words,
(a), (b) and (c) echo the thoughts and arguments of ‘harbingers of doom…’). d is inaccurate, as it finds
the author; therefore, are incorrect. refer to the line, no support in the passage.
‘individuality, it would seem, has birthed an 117. (a) the correct answer is a. the first sentence of the third
inflexibility of opinion, so much so that a difference paragraph clearly indicates that option (a) can be
in views is now tantamount to a moral inferred. refer to the lines, ‘mp has pioneered a rule
transgression.’ this is supported in option (a). ‘refer that, if replicated, can easily deal a body blow to that
to the line, ‘what has changed, according to experts, freedom. bad ideas have a way of replicating across
is a growing awareness of mental health, leading, in states – witness how many states have adopted the
turn, to deeper appreciation of how toxic or abusive self-defeating rule on private industry reserving jobs
family relationships can affect psychological well- for locals.’ option (b) is clearly in contrast to what
being.’ option (c) resonates with the lines; therefore, the author says about the film industry in india.
is incorrect. refer to the lines, ‘what an embittered, option (c) is incorrect as well. the author does not
opinionated but divided family — and the world? indicate that indian filmmakers are choosing other
the question raised in the lines is corroborated in countries for their films. option (d) is clearly in
option (d); therefore, it is incorrect. contrast with what the new rule in mp suggests.
Head Office: 127, Zone II, MP Nagar, Bhopal |+91-7676564400| https://www.toprankers.com Page 12 of 16
118. (b) the correct answer is b. the author does claim that be assessed through proper statistical distribution
some indian movies are doing well with foreign complying a model rather than a simple counting.
audiences. refer to the lines, ‘india is experiencing option (b) is incorrect since this is an extreme
some kind of a filmmaking paradigm shift as ott option. though option (c) may be inferred, it does not
productions, as well as some movie hall releases, fit in to be the main idea of the passage. the author
made here get good response from critics and does not try to explain whether analysis of various
audiences abroad…’ on the other hand, the author factors is crucial or not, before making any
strengthens the claim made in (i) in the last sentence deduction about a health issue. he has already
of the second paragraph. refer to the line, ‘the larger acknowledged it. option (d) is incorrect, as the
point is of course that the mp's decision is terrible author has not compared endemic and global health
news in a democracy…’ the author has clearly issues in the passage.
strengthened his claim in (iii) in the first sentence of 122. (d) none of the above can be concluded from the
the passage itself. refer to the line, ‘alongside passage. option (a) is an assumption; whereas,
overseeing land administration, revenue collection, option (b) is just an estimate provided in the
health, education, disaster management and other passage. option (c) is incorrect, since the number
goings-on in the district, madhya pradesh collectors 8794 is just an estimate and that too of the uk, not
will, henceforth, judge ‘entertainment’ as well.’ the entire world. therefore, option (d), none of the
119. (b) the correct answer is b. the passage suggests that the above, is the correct answer.
new rule announced in mp, if replicated’ in other 123. (b) option (b) can be inferred from the passage. the
states would kill the creative expression, but if the entire passage is based on the idea that it is difficult
new rule is restricted to taking permission for to accurately predict the number of deaths related to
shooting in highly sensitive areas, then it does not a health issue. option (a) is incorrect, as the
hamper the freedom of expression, thereby estimates of alcohol related deaths in the passage
weakening the author’s contention. option (a) has been achieved by making some big assumptions.
suggests that filmmakers are discouraged from option (c) is also incorrect, as we do not know
continuing with the filmmaking process, making it whether flumomo model is the best available model
an improbable option. the option does not or not. in fact, the author has reservation about
undermine simply because of its illogicality and flumomo model. option (d) is incorrect, as it is an
extremity. option (c) is clearly incorrect. if the incorrect inference. the author in the passage
filmmakers present scripts tailored to get a nod from addresses the very issue that one cannot single out a
the officer, creativity is bound to be compromised. specific disease, if the alcohol consumption
option (d) is a probability, as the decision by the contributed to the death. refer to the lines, ‘drinking
supreme court may go either ways; therefore, not too much alcohol raises the risks of many diseases,
really weakening the contention. so analysts try to estimate total numbers of deaths
120. (a) a is the correct answer. the author has clearly stated attributable to alcohol consumption – they modelled
in the last sentence of the second paragraph that about 19,200 alcohol-related deaths in england in
audience and talent has kept india’s film industry 2019, around four times the direct count of number
going only because of creative freedom that they of alcohol-specific registrations.’
get. therefore, it is correct to conclude that audience 124. (d) the correct answer is option (d). both statements i
and talent alone cannot keep india’s film industry and ii are assumptions made by the agency that
going. creativity and freedom of expression play a estimated the number of alcohol related deaths - not
major role too. options (b) and (c) cannot be the assumptions made by the author. therefore,
inferred, as none are supported in the passage. neither of the statements is an assumption. the rest
option (d) is clearly irrelevant in the present context. of the options are incorrect as a result.
121. (a) the correct answer is a. refer to the lines, ‘harm can 125. (a) the correct answer is a. on a close reading of the
increase without greater total consumption: the second paragraph, it is clear that this statement is an
distribution matters, not just the average.’ and, assumption made by the author. refer to the lines,
‘people might like a single number to settle an issue ‘drinking too much alcohol raises the risks of many
but we cannot count everything directly and analysts diseases, so analysts try to estimate total numbers
may need to construct a range of statistics to of deaths attributable to alcohol consumption – they
improve our understanding.’ and, ‘influenza also modelled about 19,200 alcohol-related deaths in
demonstrates the limitations of counting.’ the author england in 2019, around four times the direct count
here illustrates through alcohol related deaths, air of number of alcohol-specific registrations. the
pollution related deaths and influenza related deaths method requires many assumptions, such as 11%
that it is much more complex to explain a health of breast cancer deaths in women over 75 being due
issue through plain numbers, and requires a model to alcohol, and a major recent change in such
based on statistical approach to reach a certain attributable fractions cut estimated alcohol-related
accuracy. many deaths are not exclusive to a single death figures by around 23%.’ if the statement given
disease, but many attributable factors that can only in this question was incorrect (the negation test), it
Head Office: 127, Zone II, MP Nagar, Bhopal |+91-7676564400| https://www.toprankers.com Page 13 of 16
would have been impossible for that statement to be 128. (c) option a- this option is incorrect because the author
true. hence, it is an assumption. option (b) is nowhere talked about the online education system
incorrect, as the statement had to have further for kids.
examples to be the supporting idea. option (c) is option b- this option is incorrect as the author
ruled out, as it is the basis of the argument and id nowhere talked about the examination system
relevant as an assumption. option (d) is incorrect, directly in the passage; rather shows concern about
based on the reasoning provided for option (c). the whole education system.
126. (b) option a: this option is incorrect because it is just one option c- this option is correct; the author is stating
of the things that the author is talking about. this that there is a need to rethink the education system
statement is helping the idea of improving the and improve the learning process. refer to these lines
education system. from the passage, ‘the pandemic itself can be a
option b: this option is correct; the author is talking starting point for inquiry-based learning: children
about the opportunity to re-think and re-invent the can be encouraged, within the safe space of the
education system. you can look at the given below learning circles, to discuss what they saw,
excerpts to verify the same, ‘in the midst of experienced and learned during the pandemic.
discussions about learning deficits, remediation, karnataka’s vidyagama programme, which began
accelerated learning, and so forth, what is clear is with a group of committed teachers creating
that it should not be business as usual. education is informal, outdoor learning circles or “vataara
not a race. it is a child’s journey to fulfilling his shaale” during the pandemic - where children
potential. reopening schools should be an gathered with a teacher for in-person teaching in
opportunity to rethink the teaching and learning small groups, in outdoor community spaces; not
process itself.’ bound by blackboards and textbooks, but learning
‘rather than being cooped up inside small, cramped interactively, through stories and activities - has
schoolrooms through the day, extending lessons into been acknowledged as an example of an alternate
outdoor spaces, where feasible, will also improve model of teaching and learning.
ventilation, which is perhaps next in importance option d- this option is incorrect, as the author does
only to vaccination during the continuing not imply restructuring the education system
pandemic.’ completely. this is an extreme step which is not
‘symbolically, too, an open classroom presents itself supported in the passage.
to fresh winds and ideas. we should really open up 129. (c) options a and b- these options are incorrect, as
our education system.’ though both are correct individually, they are
option c: this option is incorrect. though the former included in option c, which is the right answer.
part of the sentence till filling the gaps in the option c- this option is correct, both a and b together
education system finds support in the passage, the make the lessons that the author talked about for the
latter portion, ‘to improvise it as per the industry set- betterment of the indian education system. you can
up is not true. look at the given below excerpts to verify the same:
option d: this option is incorrect; the author there are many lessons here for the indian education
indirectly talked about this only but the main agenda system today. joyful and creative learning should
is to make learning effective. also, it is incomplete not be an indulgence meant only for privileged
to be the main point. children. every child should be able to learn in an
127. (a) option a- this option (a) is correct. the main essence atmosphere that is free, reflective and affirming.
of the statement can be found in sentence that they should be able to relate new concepts to what
precede and follow the main sentence. refer to the they are already familiar with within their own lives.
lines, ‘for him, education meant much more than option d- this option is incorrect, as it finds no
rote learning: “the highest education is that which mention in the passage.
does not merely give us information but makes our 130. (b) option a- this option is incorrect, as the author
life in harmony with all existence.” he dismissed any nowhere talks about what children say here and
pedagogy that sought to cut children off from the want to learn.
world around them: “we are made to lose our world option b- this option is correct because according to
to find a bagful of information instead. this statement the education system will be based on
option b- this option is incorrect; as it is not giving constantly inquiring about the learning requirements
out the correct meaning of the given statement i.e., or daily learnings based on what they saw,
the author is stating that the school separates kids experienced and learned. refer to the lines, ‘the
from the actual learning which they can learn from pandemic itself can be a starting point for inquiry-
the outside world.’ based learning: children can be encouraged, within
option c and d- these options are incorrect, as clearly the safe space of the learning circles, to discuss what
this statement is completely out of context. they saw, experienced and learned during the
pandemic.’
Head Office: 127, Zone II, MP Nagar, Bhopal |+91-7676564400| https://www.toprankers.com Page 14 of 16
option c- this option is incorrect. though the intended parents are currently involved in. sentence
approach is correct, it is impossible to make changes i, however, states that the "proven infertility through
on a daily basis, and it also finds no support in the diagnosed medical conditions should indeed be the
passage. only criteria to permit surrogacy", which is a change
option d- this option is incorrect, as it finds no such in this criterion. sentence vi directly states that bill
mention in the passage. also, the approach is surely should modify itself by including new rules to make
impractical. adoption easier.
131. (b) the given stanza says "the intending couple sentence ii & iii states what would happen to enforce
meanwhile requires an official certificate to opt for the bill as it is. sentences iv & v say how the bill
surrogacy." this means that in order to get a would be violating the rights of the women or how
surrogate mother, the couple needs to avail this regressive in nature the bill is, without providing any
certificate. sentence 2 says the government itself step so change the nature of the bill. hence, the
will issue this certificate. none of the other sentences correct answer is c.
actually state any definitive actions of the 134. (a) only sentence vi supports the bill. it says that "the
government. hence, the correct answer is b. ban should be followed…." this means that it wants
statement v states a consequence of the bill, which the ban on surrogacy to be in place. sentence i also
according to the experts is a violation of the rights demands the modification of the eligibility of the
of the women. so, this statement criticises the bill. intended parents for surrogacy, but it suggests to do
making option (a) an incorrect option. so, by removing the ban itself. sentence ii & iii are
sentence i states that the "proven infertility through presenting neutral stances, as they state the
diagnosed medical conditions should indeed be the procedure of application of the bill and the impact
only criteria to permit surrogacy", which is a change of the bill, if implemented. they do not specify
in this criterion, making it an incorrect option. whether they support or criticise the bill. sentences
sentence iv claims that the bill would deny rights of iv & v say how the bill would be violating the rights
the single parents, homosexuals, transgenders of the women or how regressive in nature the bill is.
"making it a regressive or limited view of what a they are clearly against the bill. hence, the correct
family is". clearly this statement criticises the bill answer is a.
stating one of the aftereffects of the bill, making 135. (d) Sentence III states the impact of the Bill, if
option (c) incorrect, as both the sentences i and iv implemented. Sentence V mentions ‘will also take’,
incorrect. implying the possibility of occurrence of a possible
in option (d), statement iii states the impact of the event and hence, is a direct impact. Sentences IV
bill, if implemented. sentences i and iv are not true mentions how the Bill would be violating the rights
in the case; therefore option (d) is an incorrect of the women or how regressive in nature the Bill is,
option. without providing any future consequence of the
132. (d) as stated in the given stanza, the surrogacy bill Bill. Similarly, sentences I & VI provide
determines the eligibility of the intended parents for modifications to the current form of the Bill, but do
surrogacy by the nature of the relationship, the not provide any information on what would happen
intended parents are currently involved in. sentence if the bill does come into force. Hence, the correct
i however, states that the "proven infertility through answer is D.
diagnosed medical conditions should indeed be the
only criteria to permit surrogacy", which is a change SECTION - E : QUANTITATIVE TECHNIQUES
in this criterion. this makes sentence i undesirable.
sentence iv claims that the bill would deny rights of Common Explanation (Q.1-Q.5):
the single parents, homosexuals, transgenders
"making it a regressive or limited view of what a
family is". clearly this statement criticises the bill
stating one of the aftereffects of the bill. statement v
states another consequence of the bill, which
according to the experts is a violation of the rights
of the women. so, this statement criticises the bill
too.
sentence 3 also states a consequence of the bill, but
unlike statements iv & vi, it doesn’t state whether
this consequence will be a good or bad one. so, we
cannot say whether this criticise the bill or not. 136. (b) Total students in group Y of all the classes = 70
hence, the correct answer is d. 137. (a) Required percentage = 40×100/25 = 160 %
133. (c) as stated in the given stanza, the surrogacy bill 138. (a) Total girls in group Y and Z together = 34 – 12 = 22
determines the eligibility of the intended parents for Total boys in group Y and Z together on class A
surrogacy by the nature of the relationship, the = (90-30) – 22 = 38
Head Office: 127, Zone II, MP Nagar, Bhopal |+91-7676564400| https://www.toprankers.com Page 15 of 16
SECTION - D: LOGICAL REASONING
Passage (Q.106-Q.110): Ours is an era in which there is little left of nature that has not been extensively
altered by the activities of human beings. Among proposed remedies is preservation, setting aside areas
that still remain undistributed and protecting them against human encroachment, and restoration, bringing
degraded areas back to an unspoiled condition. At first thought, one might suppose that preservationists
and restorationists would make natural allies, but even a cursory reading of the relevant literature shows
that all is not harmony and peace between the two groups. The nub of the critique of preservations is the
claim that it rests on an unhealthy dualism that conceives nature and humankind as radically distinct and
opposed to each other. Dissatisfaction with dualism has for some time figured prominently in the
unhappiness of environmentalists with a mainstream industrial society, as in the writings of Carolyn
Merchant and Theodore Roszak.
However, the writings of the restorationists themselves particularly, William Jordan and Frederick Turner
offer little evidence to support this indictment. In their view, preservationists are imbued with the same basic
mindset as the industrial mainstream, the only difference being that the latter exalts humans over nature
while the former elevates nature over humans. While it is perhaps puzzling that Jordan and Turner do not
see that there is no logic that requires dualism as a philosophical underpinning for preservation, more
puzzling is the sharpness and relentlessness of their attack on preservationists, accentuated by the fact
that they offer little, if any, criticism of those who have plundered the natural world. The crucial question,
however, about the restorationist outlook has to do with the degree to which the restorationist program is
itself faithful to the first principle of restoration: that nature and humanity are fundamentally united rather
than separate.
Rejecting the old domination model, which sees humans as above nature, restoration theory champions a
model of community participation. Yet some of the descriptions that Jordan and Turner give of what
restorationists are actually up to, for example, Turner's description of humans as "the lords of creation," or
Jordan's statement that "the fate and well-being of the biosphere depend ultimately on us and our
relationship with it" do not cohere well with the community participation model. Another holistic model
namely, that of nature as an organism might be more serviceable to the restorationists. As with the
community model, the "organic" model pictures nature as a system of interconnected parts.
A fundamental difference, however, is that in an organism the parts are wholly subservient to the life of the
organism. If we could think of the biosphere as a single living organism and could identify humans with the
brain (or the DNA), or control center, we would have a model that more closely fits the restorationists' view.
However, to consider humans as the control center of the living earth is to ascribe to them a dominating
role in nature. Is this significantly different from the old-fashioned domination model? In both systems,
humans hold the place of highest authority and power in the world. Also, neither view recognizes any limits
to the scope and range of legitimate human manipulation in the world. This does not mean that there are
no constraints; only beneficial manipulation should be undertaken. But it does not mean that nothing is off-
limits. A further parallel is that, because the fate of the world rests on humans, they must have a clear idea
of what needs to be done.
106. Which of the following correctly states the author's primary purpose in the passage?
(a) Examine the similarities and differences among models for environmental philosophies.
(b) Formulate a new philosophical model of the relationship between humans and their environment.
(c) Critique a modern-day environmental philosophy.
(d) Argue that one particular environmental philosophy is more workable than competing approaches.
107. The author of the passage would probably agree that preservationists
(a) are not critical enough of those who have plundered the natural world.
(b) base their ideas on an unhealthy dualism.
(c) have the same basic mind-set as the industrial mainstream.
(d) have been unfairly criticized by restorationists.
Head Office: 127, Zone II, MP Nagar, Bhopal |+91-7676564400| https://www.toprankers.com Page 26 of 36
108. Which of the following best expresses the function of the first paragraph in relation to the passage as a
whole?
(a) To establish the parameters of an ensuing debate.
(b) To identify problem areas within a school of thought, which are then explored in greater detail.
(c) To discuss secondary issues as a prelude to a more detailed examination of a primary issue.
(d) To provide an historical backdrop for a discussion of modern-day issues.
109. In asserting that the organic model might be "more serviceable to the restorationists" the author implies
that
(a) the descriptions by Turner and Jordan of the restorationists' program conform more closely to the
organic model than to the community participation model.
(b) the organic model is more consistent than the community participation model with the principle of
restoration.
(c) the organic model is more consistent with the restorationists' agenda than with the preservationists'
program.
(d) holistic models are more useful than the dualist model to the restorationists.
110. Which of the following models would the author most likely agree is least like the other models listed below?
(a) Domination model. (b) holistic model.
(c) community participation model. (d) Dualism.
Passage (Q.111- Q.115): The President of India has sent condolences to Vandana Mishra’s family, after
her death in Kanpur late Friday night, when the car carrying her to hospital was stopped for the presidential
convoy to pass. But while the President’s response is in keeping with obligations of his high office, the
Kanpur tragedy is by no means unique. VVIP convoys have become yet another manifestation of a political
and administrative culture where public servant’s self-esteem depends on the distance they can maintain
from the public. Police are overzealous because they respond to a system where public inconvenience is
extremely low priority.
VVIP convoys in Indian cities are particularly unwelcome. Most cities suffer terrible traffic congestion. In
Mumbai it is estimated that a trip takes about 53%more time than it should, in Bangalore 51% and in New
Delhi 47%. On top of this there are few green corridors for ambulances. So, road blockades set up to
smoothen VVIP movement worsen the situation painfully. There was hope following the 2017 Union
Cabinet decision to end the lal batti raj. But long snaking VVIP convoys aren’t part of this change. And
MLA’s from Haryana to Himachal Pradesh have tried to retain traffic privileges with flags atop their vehicles.
By contrast, in America the idea of holding up citizens for hours for the convenience of politicians would be
laughter. In New Zealand, when the PM’s motorcade was caught over the speed limit, it was fined for
dangerous driving. In Scandinavian countries, far from reserving a faster lane for themselves, netas take
public transport shoulder to shoulder with citizens. This is the direction our democracy must take too. For
VVIP protection, the state must deploy smarter security arrangements, and nix those convoys. When public
servants are driven, let the public not be driven to despair or worse.
111. Which among the following is the most significant message conveyed by the author of the passage?
(a) The VVIP system is not suitable for India since it causes inconvenience to common people.
(b) A public servant’s self-esteem carries a lot more significance than that of a common man.
(c) It is time that the state does away with convoys and brings up a smarter security system for VVIPs.
(d) The President should be held responsible for the death of Vandana Mishra.
112. Which of the following courses of action should India take from the various countries mentioned in the
passage?
I. Hold up citizens for long hours for politicians.
II. Levying a penalty on the vehicles of the politicians for violating traffic rules.
III. Oblige politicians to use public transport.
(a) Only I (b) Only II (c) Both II and III (d) All I, II and III
Head Office: 127, Zone II, MP Nagar, Bhopal |+91-7676564400| https://www.toprankers.com Page 27 of 36
113. Which of the following assumptions must be true?
(a) The tragedy that happened due to the convoy of the President is one of the last such tragedies.
(b) At least one of the political and administrative VVIP manifestations should be done away in India.
(c) Deploying smarter security systems for VVIP convoys is a one-stop solution for the perils of
democracy.
(d) VVIP convoys should not be done away with in a democracy such as India.
114. While giving examples of other countries, which among the following is the central flaw made by the author?
(a) The traffic situation in Indian and Western countries is broadly similar.
(b) All countries being mentioned are Europeans.
(c) Time is the essence of life.
(d) Politicians should be held responsible for their actions.
115. ‘In Scandinavian countries, far from reserving a faster lane for themselves, netas (political leaders) take
public transport shoulder to shoulder with citizens.’ Which of the following statements, from the options,
would weaken the argument?
(a) The political leaders have been put on a pedestal by the people who wish to be in awe of their leaders.
(b) The political leaders are known for their indifference towards development and people may vent out
their anger on the leaders, if the leaders were to mingle with people.
(c) The leaders being the representative of people face constant death threats and are under the radar of
those who want to sabotage the development, cannot be treated as commoners as they hold position
of relevance.
(d) The leaders are representative of people and conducting themselves with humility will only strengthen
their support amongst their people.
Passage (Q.116-Q.120): Research shows that intergenerational shared sites increase the health and well-
being of both young and older participants, reduce social isolation, and create cost efficiencies. They are
joyful places. And unsurprisingly, the concept isn’t unpopular. Americans are overwhelmingly in favor of
shared sites. Shared sites benefit and build the health-care workforce as well. One such plan operates a
high-school program that has graduated nearly 900 at-risk students. The program has not only sparked
student interest in health-related careers but also helped them develop a range of useful life skills. Finally,
attracting and retaining staff—a chronic problem in the care industry—is less of an issue at shared sites,
because employees can more easily meet their own family caregiving needs, and many find joy working
with both children and seniors.
And yet, while successful shared sites exist in countries around the world, there are fewer than 150 of them
in the United States, compared to tens of thousands of age-segregated care facilities around the country.
The answer is that this relatively simple care model, it turns out, is difficult to pull off, complicated by funding
silos and cumbersome regulations. Most funders that support care facilities focus on either childcare or
eldercare, forcing operators of shared sites to seek and manage separate funding sources. Furthermore,
the staffing plans, emergency evacuation procedures, and other regulations that govern care sites for
younger people differ from those designed for older populations.
To solve this problem, the first step is increasing awareness. Simply making intergenerational connection
more visible through the design of buildings, community engagement, and promotional materials is one
way to do this. Further, a shared site that draws on the talents and resources of the local community, and
that invites a range of groups to take part in its successes and tribulations will more likely prosper over the
longer term.
Head Office: 127, Zone II, MP Nagar, Bhopal |+91-7676564400| https://www.toprankers.com Page 28 of 36
116. Which of the following is most supported by the author’s argument?
(a) As the building plans required for eldercare is extremely different from the same being required for
the childcare, shared sites can’t be a good idea always in the society.
(b) The staffing required for shared sites are easy to get as lots of people have showed their interest in
joining these organisations and owing to the popularity and unique cross-generational experience it
attracts fair interest among the youth.
(c) Though the concept of shared sites is a popular one, its advantages and conveniences need to reach
the people to a much larger extent than what it is presently
(d) Both B and C
118. As per the author, why are shared sites a good initiative?
(a) Shared sites have allowed the government to solve two purposes with a single operation – taking
care of the elders as well as the children in one single space. This is huge problem that has been
solved.
(b) Instead of investing in two areas, the investor invests just for one centre, which means there is a huge
scope of savings that generates from these centres.
(c) The health care benefits and workforce gets built form the shared sites, creating an awareness of
careers for the younger generation.
(d) The staffing plans, emergency evacuation procedures, and other regulations that govern care sites for
younger people differ from those designed for older populations.
119. Which of the following, if true, would most seriously undermine the author’s argument?
(a) Studies suggest that young children are incapable of being very friendly and are prone to violence
towards people elder to them.
(b) Some senior citizens do not have the energy and mental faculties to entertain and engage younger
people.
(c) Majority of working people prefer to send their aged parents to old age homes rather than keeping
them at home.
(d) The interaction between the older and the younger generation can prevent mental and emotional
disturbances prevalent amongst the youth.
120. The author’s statement that “Research shows that intergenerational shared sites increase the health and
well-being of both young and older participants” forms the:
(a) premise of the above passage. (b) conclusion of the above passage.
(c) assumption of the author. (d) It is just a general statement.
Head Office: 127, Zone II, MP Nagar, Bhopal |+91-7676564400| https://www.toprankers.com Page 29 of 36
Direction (Q.121–Q.125): India is one of the largest producers of food grains globally. Though its
agriculture is very small-scale, the yield from farming is low and the pressure on land is enormous. This
needs to change with the help of digital precision technology. India’s agricultural policies have seen a major
boost since the Modi government came to power in 2014 and decided to focus on doubling farmers income
by 2022. The government has also set an ambitious food grains production target of 291.1 million tons for
2019-20, an increase of 2.6% compared to the previous year, citing a favourable monsoon in the current
season. The government has chalked out plans to achieve the goal; from agriculture productivity, soil health
cards, crop insurance, irrigation, total mechanization, technology, animal husbandry and allied activities.
But is it on the right track, many ask. The Consortium of Indian Farmers Associations (CIFA), India’s apex
professional farmer’s organization allege ineffective policy level interventions on behalf of farmers from all
over India. It said recently that to double farmers income, the government needs to implement a consistent
export-import policy and proactive market intervention to attain fair prices for agricultural produce.
122. Which of the following may best explain the term ‘animal husbandry’?
(a) A controlled cultivation, management, and production of wild animals.
(b) Animals bred and raised for consumption.
(c) A controlled cultivation, management, and production of domestic animals.
(d) Animals bred and raised for sport, and research.
123. If the information contained in the passage is true, then it is also true that
(a) India needs a different set of solutions for agriculture and for those working the land.
(b) India needs to shift from basic farming to more efficient, sustainable, and productive farming.
(c) Today’s agriculture policies fail to recognise how crop choices, input costs, and the supply chain are
intertwined, perpetuating marginal farming.
(d) However, even if we doubled or tripled our output, would that double or triple farmer’s earnings? No,
since a glut would reduce prices.
125. Which of the following statements discloses the intention of the writer?
(a) To emphasize the need to implement CIFA’s agricultural policies and rules.
(b) To laud the actions of the BJP government in achieving greater heights in Indian production and income
of the farmers.
(c) To highlight the need to focus on ambitious and proactive advances in the fields of Indian agriculture.
(d) To show concern over the overambitious goals of the Modi government.
Head Office: 127, Zone II, MP Nagar, Bhopal |+91-7676564400| https://www.toprankers.com Page 30 of 36
Passage (Q.126-Q.130): Death is in the air. According to a recent study by ‘The Network’, C40, among 61
global megacities, Calcutta recorded the highest number of premature deaths on account of air pollution
caused by coal-fired thermal power plants in 2019. It is also projected to witness 10,000 premature deaths
by 2030. Among Indian cities, it is followed by Mumbai. This is not surprising. In spite of much-lauded
efforts in clean energy, the carbon footprint of India’s coal-processing industry remains stark. The country
has the world’s second highest number of operating coal power plants, none of which is scheduled to be
phased out by 2030. In fact, a draft of the National Electricity Policy 2021 revealed plans to continue building
coal-fired plants as they provide a cheap source of power. The rampant use of dirty coal in existing plants
only compounds the problem — 84 per cent of the power stations supplying electricity to Bengal do not
comply with official regulations for sulphur dioxide emissions. ‘Clean coal’ alternatives are not viable
solutions either, since their contributions to greenhouse gas emissions and air pollution are high. Clearly,
a move away from coal as a whole is necessary.
Policies, however, continue to be guided by false assumptions. While rural households are believed to
contribute majorly to coal consumption, coal accounts for about 70 per cent of India’s electricity generation,
consumed at a much higher rate in urban centres. What available data suggest is a worrying gap between
India’s commitment to adopt clean energy sources and the reality on the ground. To meet the 2030
emissions goal committed at the Paris Pact, India would need to reduce coal capacity in and around its
C40 cities by about 22 per cent. Yet, going by the current coal plans, it is expected to increase by 20 per
cent instead. To bridge this gap, which is unlikely, policymakers need to look beyond populist steps. While
the Ujjwala scheme promises free liquefied petroleum gas — an environment-friendly alternative to coal —
to poor households, the government seems oblivious to the plight of the Indian middle class, struggling to
afford LPG because of soaring prices. Investments in renewable energy need to be increased manifold —
it constitutes just 33 percent of the country’s recent spending on energy infrastructure. Populism must be
junked and changes in public lifestyle encouraged in policy for India to cleanse the air.
126. Which among the following can most likely be inferred from the passage?
(a) The policy makers tend to appeal to the ordinary people while making decisions on coal.
(b) The Ujjwala scheme caters to the demands of all the economic groups of the country.
(c) India has a proven track record on keeping the promise it makes, especially about environment
protection.
(d) None of the above can be inferred from the passage.
127. Which of the following can be considered as an opinion of the author? Choose the best answer.
(a) The author is bewildered by the findings of ‘The Network’.
(b) The author is not concerned with the data released by ‘The Network’.
(c) The author believes that the data released by ‘The Network’ is not credible.
(d) The author agrees with the findings of ‘The Network’.
128. Out of the following, which one would be considered a logical course of action by the author?
i. The government takes measures to keep the prices of the Liquefied Natural Gas under control,
especially for the middle class.
ii. The government focuses on obsolescence of coal in the market to reduce the pollution levels.
iii. The policy makers, henceforth, make genuine attempts at providing the needed solution and do not
take up a policy to please the majority.
(a) Only I (b) Only I and II (c) Only I and III (d) All of the above
129. Which of the following is not likely to be inferred of the passage? Choose the most appropriate option.
(a) India would reduce its coal capacity in and around its C40 by 20 percent.
(b) The policy makers should make sure that it makes policy decisions based on credible pieces of
evidence.
(c) India is unlikely to meet its 2030 emissions goal committed at the Paris Pact.
(d) India has made commendable progress in the industry of clean energy.
Head Office: 127, Zone II, MP Nagar, Bhopal |+91-7676564400| https://www.toprankers.com Page 31 of 36
130. Consider this statement, ‘A transition to 100 per cent cleaner and greener energy could generate 18-27
per cent more productivity across India.’ Which among the following roles would this piece of evidence, if
true, plays towards the arguments of the author?
(a) This piece of evidence would support the author’s arguments.
(b) This piece of evidence would weaken the arguments made by the author.
(c) This piece of evidence is unlikely to play a role towards the author’s arguments.
(d) This piece of evidence is likely to be an assumption made by the author.
Passage (Q.131–Q.135): The decision by ASEAN to exclude Myanmar’s military junta from its annual
summit held on October 26-28 is a major setback for the Generals’ attempts to gain regional legitimacy for
their brutal regime. Ever since it seized power by toppling the democratic government of Aung San Suu
Kyi in February, the junta has unleashed a reign of terror claiming an estimated 1,000 lives. Ms. Suu Kyi
has been in detention since the coup and is facing absurd charges such as “illegally owning walkie-talkies”.
Thousands of others were arrested by the military, notorious for its reprisal of democratic protests in the
past. But this time, the crisis seems much worse. Months after the seizure of power, the junta, led by Gen.
Min Aung Hlaing, is still struggling to restore order. If in the past the National League for Democracy (NLD),
Ms. Suu Kyi’s party, had upheld non-violence even in the face of repression, this time, NLD leaders have
called for a “revolution”. The remnants of the old regime have formed a National Unity Government, which
claims to be the true representative of Myanmar. In cities, protests slid into armed fighting between pro-
democracy protesters and security personnel, while in the jungles, anti-junta groups joined hands with
rebels for military training. The situation was so grave that the UN Special Envoy warned this month that
Myanmar had descended into a civil war.
One of the regional groupings with some leverage over the junta is ASEAN. In April, Gen. Min Aung Hlaing
was invited to Jakarta for emergency talks with ASEAN members. The bloc asked him to immediately end
violence, start the reconciliation process and allow a regional special envoy to meet with all stakeholders,
including Ms. Suu Kyi. None of these requests was met. A special envoy was appointed as part of the
ASEAN plan, but he was not allowed to meet Ms. Suu Kyi. Regime violence, political crises and strikes and
counter-attacks by protesters have all pushed Myanmar to the brink of collapse. Violence might allow them
to hold on to power for now, but that is not sustainable. The ASEAN snub is a reminder that continuing
violence could cause regional isolation of the regime, which could worsen the crisis. The international
community should continue to put pressure on the junta and urgently start a reconciliation process.
131. Which among the following sentences from the passage is the best representation of the central idea of
the passage?
(a) The ASEAN snub is a reminder that continuing violence could cause regional isolation of the regime,
which could worsen the crisis.
(b) Most recent reports suggest that the junta has been systematically torturing political prisoners.
(c) Months after the seizure of power, the junta, led by Gen. Min Aung Hlaing is still struggling to restore
order.
(d) Ms. Suu Kyi has been in detention since the coup and is facing absurd charges such as “illegally owning
walkie-talkies”.
132. Which of the following can be validly inferred from the passage?
(a) The stance of the National League for Democracy on the military junta has remained firm.
(b) The ASEAN does not have sufficient control over Myanmar’s military junta.
(c) The situation in the forests of Myanmar is better than that of the cities.
(d) Aung San Suu Kyi is the sole person who can curb the military junta.
Head Office: 127, Zone II, MP Nagar, Bhopal |+91-7676564400| https://www.toprankers.com Page 32 of 36
133. What could be attributed to be the author’s opinion, according to the passage?
I. The snub of the ASEAN is a reminder for the military in Myanmar to leave the country, otherwise face
consequences.
II. The ASEAN snub is not likely to play any significant role in Myanmar’s military coup.
III. The military this time has cracked down on its protesters more brutally than what it did in the past.
IV. The leaders of the National League for Democracy in the second time round of the coup by military
junta are not up for a non-violence protest.
(a) Only I (b) Only II (c) Only III (d) Only IV
134. Which among the following is a future possibility of the military rule in Myanmar?
(a) The military rule in Myanmar is likely to accommodate peaceful protests.
(b) The military rule in Myanmar is likely to try to suppress any resistance.
(c) The military rule in Myanmar is likely to give up its rule and let democracy prevail.
(d) The military rule in Myanmar is likely to treat its political prisoners humanely.
135. The international community should continue to put pressure on the junta and urgently start a reconciliation
process.’ The author through the passage does which of the following?
(a) This statement is not likely to play any significant role in the formation of the passage.
(b) This statement is likely to weaken the arguments made by the author.
(c) This statement is an attempt by the author to make a suggestion.
(d) This statement is an assumption made by the author.
Head Office: 127, Zone II, MP Nagar, Bhopal |+91-7676564400| https://www.toprankers.com Page 33 of 36
database, and the other one is to still not 107. (d) The correct response is (D) because in the first
mentioning the fact of his acquittal in the paragraph, the author states that a preservationist
database. Losing the database earlier is another need not have a dualist view, and therefore the
fault on the part of the police, due to which the argument of Turner and Jordan that the
acquitted persons can suffer. preservationists are also "unhealthy" dualists is an
Option (a) is incorrect because however much time unfair claim. (D) is also supported later in the first
has it been since his acquittal, the criminal record paragraph, where the author criticizes Turner and
database must have accurate information, or it could Jordan for the "sharpness and relentlessness of their
hamper or influence the results of other cases which attack on preservationists." (A) confuses the
would be decided on the basis of the database. information in the passage.
Option (b) is incorrect because even though there is The author suggests that it is the restorationists such
no case at present, there can be a case in the future, as Turner and Jordan (not the preservationists) who
and even without that, the database must have are not critical enough of those who have plundered
accurate information, without additions or omissions, the natural world.
as can be deduced from the theme of the passage. (B) and (C) confuse the author's viewpoint with the
Option (d) is incorrect because it is totally viewpoint of others mentioned in the passage. It is
superfluous and it can be deduced from nowhere in the restorationists, not the author, who claim that the
the passage that a learned man should be treated preservationists base their ideas on an unhealthy
differently. dualism and who suffer from the same mind-set as
104. (c) Option C is correct because nowhere is it the industrial mainstream.
mentioned that Harishad was acquitted, still the 108. (c) The correct response is (C). (A) runs contrary to the
database is showing that he had been acquitted. passage and does not identify which issues are
This may lead to freeing an offender, which is subject to debate and which are not. (B) is only
again injustice on the society due to faults in partially supported in the latter portion of the
maintaining the database. statement; the former (to identify problem areas
Option A is incorrect because that Harishad has within a school of thought) does not form part of the
been acquitted has not been mentioned anywhere first paragraph. (D) distorts the information in the
and thus, it cannot be deduced. passage. Although the author includes some
Option B is incorrect because the passage does not "historical" background but apart from this single
talk anything about the time period of a database reference to past events, the paragraph speaks in
making it defunct. A 20 year old database might also terms of the present day.
be used today for decision of certain cases. There is 109. (a) The correct response is (A). The author states that
no bar to that. Turner's and Jordan's descriptions of restorationist
Option D is incorrect because it is the responsibility activities "do not cohere well with the community
of state to maintain correct database. Be it of any use participation model." On following this, it is
or not but information has to be correct and updated. reasonably inferable that restorationists' activities
Even if in present situation there is no another case are more consistent with this other model than with
against him but it does not eliminate the possibility of the community participation model.
another case in future. (B) confuses the information in this portion of the
105. (a) Option A is correct because all the facts passage. It is contrary to what has been said in the
mentioned in option A are given in 1st line of the passage. (C) confuses the information in the
1st para in the passage. passage-specifically, by bringing in irrelevant
Option B is incorrect because the passage says information. The author is not concerned at all in this
Article 21 would be violated and not Article 20. portion of the passage with the preservationists. (D)
Moreover, the Delhi Court judge mentioned Delhi is somewhat consistent with the information in the
police to update the database and not the Bihar passage, but it does not respond to the question. The
police. author does identify the organic model as one type
Option C is incorrect because a Delhi Court judge of "holistic" model, but, the author states that it may
said the same, instead of a Bihar Court judge. be more serviceable than another holistic model.
Option D is incorrect as the passage says Article 21 110. (d) The correct answer is (D). The author finds some
would be violated and not Article 20. point of similarity among all other models mentioned.
Therefore, by elimination, (D) is the best response.
SECTION - D : LOGICAL REASONING Refer to the lines, ‘Dissatisfaction with dualism has
for some time figured prominently in the unhappiness
106. (c) Although the passage does digress in the last of environmentalists with a mainstream industrial
paragraph (suggesting a possible transition to society…’
another area of discussion), the passage is devoted (A) and (B) are not viable. The author points out
mainly to a critical analysis of the restorationists' several similarities between the organic model (a
environmental philosophy, as exemplified by Turner holistic model) and the domination model. (C) is not
and Jordan. A is too narrow. The author makes no a viable response, since the author points out in the
claim to having formulated the organic model or that paragraph that the community participation and
it is a "new philosophical model”. B is too narrow and organic models both picture nature as a system of
not well supported. (D) distorts the author's purpose. interconnected parts.
(C) supports well the primary purpose of this 111. (c) Option C is the correct answer because the author
passage. tries to put forth through the passage to remove the
Head Office: 127, Zone II, MP Nagar, Bhopal |+91-7676564400| https://www.toprankers.com Page 9 of 17
VVIP convoy system and enable a more intelligent weaken the argument is to present a counter
security system that will benefit the public. Option A argument that completely invalidates the reasoning.
is not correct as it is not about only India but for any Option (c) weakens the argument completely. If the
country. It will provide a hurdle and inconvenience to leaders, who are the representatives of the people
the ordinary person and pose a threat to them. and have important works of national interest are
Option B is not correct, as it is not the message that under threats, they cannot be exposed in the public.
the author wants to pass on. Option D is not valid as Hence, they need protection, for which they have to
only the President cannot be held liable for the death maintain distance and cannot be seen in public
of Vandana Mishra as the VVIP convoy system was transports. Option (b) may appear close by way of
not implemented by him. their indifference and people may vent out their
112. (b) Statement I is contrary to the essence of the anger is to assume that all leaders are indifferent,
passage. Statement II is correct as it is the desirable which is an extreme case. Option (a) and (d)
course of action, as per the author. Refer to the lines, strengthen the argument. In option (a0, if people are
‘In New Zealand, when the PM’s motorcade was in awe (marvel), they do not want their leaders to
caught over the speed limit, it was fined for conduct themselves as commoners, but would like to
dangerous driving.’ Statement III is not the correct know them from afar and they are fine with the
course of action for India, given the size of the distance. Option (d) is actually the inference of the
population and a smooth working of the functionary. main argument.
A course of action should be practical and feasible, 116. (d) Options (A) can’t be supported as though the
keeping in mind the place and the factors involved. passage mentions that the building and logistics are
One country’s asset can be other country’s liability. difficult to get and different for the elder-care and the
Option B is the correct answer as Article 14 of the child-care and mixing them up in a shared site makes
Indian Constitution states that the law holds every it a lot more difficult, he doesn’t say that this makes
person to be the same before it, and no one acquires it a bad idea for the society.
a higher position in the eyes of the law. So a politician Option (B) can be supported as the author says that
doesn’t have any right to violate traffic rules, just like this concept is actually a pretty popular one and also
an ordinary man. Option A is incorrect as statement lots of students have showed interest in joining these
I is not the right course of action. Option C is incorrect organizations.
as Statement III is incorrect and Option D is incorrect Option (C) can be supported as the author says that
as both Statements I and III are wrong courses of there are many impediments to the concept of
action. shared sites, but more awareness can negate those.
113. (b) Option B is the correct answer as if any one of the Thus, this option too is correct.
political and administrative VVIP manifestations is Thus, the answer is option (D)
not done away, the people of India will continue to 117. (c) Option (A) can’t be inferred from the paragraph as
suffer, and their rights for equality will be infringed. the author says that people are not that in to
Option A is not correct as it is not relevant information investing in these sorts of spaces. Also, tax-free has
in the passage, and, therefore, not the right not been mentioned anywhere.
assumption. Option C is not valid, as bringing more Option (B) can’t be inferred as it has not been
innovative security into action will not end the perils mentioned. The paragraph says that it rises interests
of democracy; therefore, not the correct assumption. among the workers and employees of the shared
Option D is not correct, as VVIP convoys should be sites, but whether the elders prefer to live in these
away within a democracy such as India; otherwise, it centres has not been made clear.
will differentiate the citizens of India. Option (C) can be inferred from the paragraph as the
114. (a) A central flaw is that the argument mistakes a author says that in US, people must be made aware
condition sufficient for bringing about a result for a of its benefits. Option (D) cannot be inferred as it is
condition necessary for doing so. Option A is the in contradiction to the information in the passage.
correct answer as the author while putting forth the The sites do not increase costs; rather are cost
argument assumes the traffic situation in India and efficient. Also, isolating the health care workers form
Western countries to be the same (a condition critical care is not mentioned anywhere in the
sufficient to bring about the changes. Assuming passage.’
similar situations and overlooking other factors in Thus, the answer is option (C).
reaching an outcome). Option B is the incorrect 118. (c) Option (A) presents a fact that has not been
answer as it is not irrelevant because all the mentioned by the author. It is actually difficult to build
countries mentioned in the passage are Europeans. places which is suitable for both the children and
Option C is not the correct one, as this is the truth elders and that is why these centres are low in
under all circumstances and is not a central flaw. number.
Option D is not valid as the situations, and the Option (B) again is a hypnotized point mentioned.
treatment for the public servants in India and the The author says that the investors have shied away
Western countries are not the same. Also, this is the from investing in such centres and is exactly the
conclusion one can draw, but cannot be said to be opposite of the option. Option (C) is relevant as the
the central flaw. answer as it accurately addresses the benefits of the
115. (c) The argument above talks of the conduct of the intergenerational share sites. Refer to the lines,
political leader whereby they do not take advantage ‘Shared sites benefit and build the health-care
of their position, but are true representatives of their workforce as well’ and ‘‘The program has not only
people. The main argument is to be weakened. To sparked student interest in health-related careers but
Head Office: 127, Zone II, MP Nagar, Bhopal |+91-7676564400| https://www.toprankers.com Page 10 of 17
also helped them develop a range of useful life skills.’ correct terminology; hence, are unsuited as
Option (D) poses the challenges of the shared sites, answers.
instead of the advantages. 123. (b) The entire passage talks of Modi Government’s
Thus, option (C) is the answer. vision and ambitious plans of doubling farmer’s
119. (a) Option (A) provides a valid problem. The centres income along with food grains production target of
such as these are reliant on good social behaviour 291.1 million tons for 2019-20, an increase of 2.6%
from the elders as well as the children. If there is a compared to the previous year. For this the
roadblock from either side, it can be a genuine government has chalked out ways to do this. Option
problem in popularising the centres. (b) is the most desirable tactical solution to achieve
Likewise, option (B) presents a similar problem, but the goal. Option (a) cannot be the furtherance of the
does not completely undermine the author’s passage as it finds no support in the passage. What
argument, as ‘some’ is a figure that is vague; in fact, different set of solutions are needed? Options (c) and
it can represent a number that does not really pose (d) take a negative tone, almost questioning the
a problem at all in the larger context. Thus, it does plan’s feasibility, rendering them unsuited as
not completely undermine, making it not a valid answers.
answer. 124. (d) It is apparent as all the other options are correct
Only Option (C) is immaterial to the discussion posed which makes Option D as correct. Refer to the lines,
by the author. It has got no relevance whatsoever to ‘The Consortium of Indian Farmers Associations
what we are talking about. Option (D) strengthens (CIFA), India’s apex professional farmer’s
the author’s argument as it is in sync with the organization engaged ineffective policy level
passage. interventions on behalf of farmers from all over India,
Therefore, the correct Option is (A). has said recently that to double farmers income, the
120. (a) Premises are the facts or evidence that support or government needs to implement a consistent
lead to the conclusion. Therefore, the above export/import policy and proactive market
statement forms premise of the passage which leads intervention to attain fair prices for agricultural
to the conclusion. Hence, option (A) is the correct produce. Options (a) and (b) can easily be deduced
answer. Conclusions are the wrap-ups of an form the lines. Option (c) is an inference. It CIFA is
argument, the expression ‘research shows’ is not a India’s apex professional farmer’s organization, then
conclusion. The above statement demands an it is clear that it is protecting the interest of the
extension of the main argument; hence, cannot be farmers. All of the following options are correct.
the conclusion. An assumption is the basis of an 125. (c) Option (c) is supported by the author, in gist and in
argument. The expression ‘the research shows’ rules essence. Option (a) is a suggestion to the
out an assumption. An assumption would be: government and not the need or a compulsion.
Researches lead to certain conclusion. Option (D) is Option (b) is partially true, but the main intention of
ruled out that a simple statement can be a premise, the author. The author more than just praises the
assumption or a conclusion. At the least, it will be a actions of the government. He is a little concerned
fact or a point of view on how such an ambitious project will be
121. (a) Refer to the lines, ‘It is apparent as all the other implemented. Option (d) is far-fetched. The author
options are incorrect. Option B is correct because it does not express that the plans of the Modi
justifies the statement, ‘The Consortium of Indian government are overambitious.
Farmers Associations (CIFA), India’s apex 126. (a) The correct answer is (a). The author has mentioned
professional farmer’s organization engaged that policymakers need to look beyond the populism
ineffective policy level interventions on behalf of steps, thereby implying that the policymakers should
farmers from all over India, has said recently that to refrain from appealing to the ordinary people. Hence,
double farmers income, the government needs to this can be inferred. Option (b) is clearly incorrect
implement a consistent export/import policy and since the Ujjwala scheme does not cater to the
proactive market intervention to attain fair prices for demands of the middle class. Option (c) is also
agricultural produce. From the lines, we can infer that incorrect since the author does not present any piece
the ambitious plans of doubling farmer’s income of evidence regarding this claim. In fact, the author
requires proactive measures form the Government’s has said that India’s goal for 2030 is unlikely to be
side and the efforts to implement export/import policy achieved, making this statement incorrect. Since
and proactive market intervention to attain fair prices option (a) is the answer, option (d) is ruled out.
is not an easy task. Option (a) can be inferred from 127. (d) The author says that he is not surprised by the
the passage. Option (b) may appear correct, but for findings of ‘The Network’ since the carbon footprint
the word ‘miniscule’, which derails the statement. of India’s coal-processing industry remains severe.
Option (c) is incorrect as which devious plan have we By saying that he is not surprised, he means that he
inferred from the passage? Option (d) is contrary to agrees with the findings as if almost expected the
the passage. (nadir?) numbers to be true. Therefore, the correct answer is
122. (c) Option (c) sums the terminology in the best possible (d). Option (a) is contrary, as bewildered means
way. Animal husbandry is a controlled cultivation, surprised, as if unexpected. Option (b) may seem
management, and production of domestic animals, correct, but the author is not unconcerned by the
including improvements of the qualities considered findings. This is because the author presents some
desirable by humans by means of breeding. Option suggestions to improve the current situation. Option
(a) is wrong as the wild animals are not involved in (c) is ruled out because had it been the case, the
animal husbandry. Option (b) and (d) are partially author would not have cited ‘The Network’s findings.
Head Office: 127, Zone II, MP Nagar, Bhopal |+91-7676564400| https://www.toprankers.com Page 11 of 17
128. (d) The correct answer is (D). All of the above can be 133. (d) Only IV can be inferred from the passage. Refer to
considered as a logical course of action, according the lines, ‘If in the past the National League for
to the author. I: The author is concerned about the Democracy (NLD), Ms. Suu Kyi’s party, had upheld
high LNG prices for the middle class. Therefore, the non-violence even in the face of repression, this
author would want the government to take measures time, NLD leaders have called for a “revolution”. The
to keep those prices under control. II: The author has remnants of the old regime have formed a National
clearly mentioned in the last sentence of the first Unity Government, which claims to be the true
paragraph that a move away from coal as a whole is representative of Myanmar. In cities, protests slid
necessary. III: This one would also be a welcome into armed fighting between pro-democracy
decision since the author wants the policymakers to protesters and security personnel, while in the
avoid making populist decisions. All of the following jungles, anti-junta groups joined hands with rebels
are the course of action that are concurrent with the for military training.’ Option (d) is the best choice as
author’s line of thought. the answer. Both I and II are incorrect since they are
129. (a) The correct answer is (a). This question is easy. As extreme options. The author believes that the
stated in the explanation of a previous question, the ASEAN snub would have some impact. But both I
author has mentioned in the last sentence of the first and II are extreme options. Statement III is beyond
paragraph that a move away from coal as a whole is the scope as it cannot be inferred that the military
necessary. But option (a) draws an incorrect crackdown is more brutal than before.
inference. Refer to the lines, ‘To meet the 2030 134. (b) The correct answer is B. Since the military has
emissions goal committed at the Paris Pact, India already killed 1000 people and brutally cracked down
would need to reduce coal capacity in and around its on its protesters, it can be safely said that it would
C40 cities by about 22 per cent.’ Option (b) , (c) and suppress any resistance by the people to its military
(d) can be inferred to be the supporting ideas of the rule. A is clearly in contrast with B and hence, is
passage. Option (b) comes from the first part of the incorrect. C is incorrect since no evidence is present
second paragraph. Policies are made on some in the passage that suggests this. D is incorrect as
unfounded assumptions and therefore this option going by their inherent characteristics, military junta
makes sense. Option (c) is a supporting idea as well, is least likely to show humane treatment of political
coming from this part of the passage: ‘Yet, going by prisoners.
the current coal plans, it is expected to increase by 135. (c) The author of this passage has clearly made a claim
20 per cent instead. To bridge this gap, which is that violence is not sustainable. And he suggests that
unlikely, policymakers need to look beyond populist the way to bring the country out of the crisis is to for
steps.’ Option (d): ‘In spite of much-lauded efforts in ASEAN to start put pressure on the junta for a
clean energy, the carbon footprint of India’s coal- reconciliation. The author attempts to make a
processing industry remains stark.’ suggestion towards the end. Therefore, Option (c) is
130. (a) The author has made an argument for shifting away the best choice. Option (a) is incorrect. The
from coal and moving towards cleaner energy. This statement has to be significant to be put forth by the
piece of evidence gives an incentive towards making author. Why else would he make such a statement?
a change to cleaner and greener energy sources. Option (b) is not weakening the argument, but is
Hence, this option supports the author’s arguments. more of a furtherance of his opinion. Option (d) is not
Therefore, the correct option is (a). Option (b) is an assumption. Assumption is not explicitly stated,
incorrect as it does not negate but strengthens the but is implied by the author.
author’s contention. Option (c) is not true as it does
support the author’s argument as a 100 percent SECTION - E : QUANTITATIVE TECHNIQUES
cleaner and greener energy translates into a clean
environment which means robust health yielding 136. (c) Total freelancers in colony 1 = 81
more productivity. Option (d) is not an assumption as So, 15% shifted to colony 3,
it is a furtherance to author’s argument. Then, total number of people in colony 3 = 200 +
131. (a) The correct answer is A. This option is best suited 80 ×15
130 + 50 + = 392
100
among the four options to be the main idea of the
137. (a) Total unemployed in all the colonies together =
passage. It talks about the main concern of the
100 + 80 + 50 + 70 = 300
passage which is the implication of the snub by the 300
ASEAN of the Myanmar Military rule. All the other Required average = = 75
4
three options do not cover the general main idea of 138. (b) Total unemployed people in colony 4 = 70
the passage but only cover some specific parts. After 20% get jobs, number of fulltime job
132. (b) The correct answer is B. This can be inferred from 70×20
employers in colony 4 = 300 + = 314
the fact that none of the requests or the directions 100
Required difference = 314 − 220 = 94
given by the ASEAN were followed by the military
139. (c) Number of fulltime employees in colony 3 = 200
junta in Myanmar. A is incorrect since the stance of
Total people in colony 1 = 300
NLD on the military junta has changed from calling (300−200) 1
for nonviolent protests to a ‘revolution’. C is incorrect Required percentage = × 100 = 33 %
300 3
since the situation both at the cities and the jungles 140. (b) Total people in colony 2 = 450
is grave. D is incorrect inference as Aung San Suu Total people in colony 4 = 550
Kyi may have hold on the people, but she has no Req. ration =
450
= 9 ∶ 11
control over the military junta. In fact, she is under 550
arrest by them.
Head Office: 127, Zone II, MP Nagar, Bhopal |+91-7676564400| https://www.toprankers.com Page 12 of 17
SECTION - D: LOGICAL REASONING
Passage(Q.106-Q.110): It is unfortunate that yet another important bill with far-reaching implications for our
democracy has been passed by both Houses of Parliament without a discussion. The Election Laws (Amendment)
Bill 2021, which provides for the linking of electoral cards with the Aadhaar number, was passed by a voice vote
amidst protests from the Opposition. Some Opposition members demanded that it be referred to a parliamentary
panel. Since the provisions of the bill related to serious matters like citizenship, voting rights and personal
identity, there should have been a detailed debate on its provisions. The government seemed to be in a hurry and
the demand for referring the bill to a parliamentary committee was rejected. Law Minister Kiren Rijiju, who
introduced the bill, told the Lok Sabha that some proposals which are part of the bill had already been
recommended by the Standing Committee of Law and Personnel. This was an inadequate reply and there was a
need for more clarity on the matter.
The bill raises some genuine and serious concerns. A voter ID card is given only to citizens; an Aadhaar number
is only a tool to verify identity, and is available to all residents, including non-citizens. A linkage between the
two is unnatural and problematic. It has been pointed out that it can potentially lead to a situation in which non-
citizens may get the voting right or bona fide citizens may be denied the right. Some critics have mentioned the
possibility of disenfranchisement of voters with the help of the new law. Electoral registration officers can
demand the Aadhaar number of those who seek a voter ID and even of those who already have the card. Refusal
can mean denial of the voting right, though the linking is now said to be optional. But if such linking was optional,
then why make a law?
The bill also faces criticism on the ground that it violates the Supreme Court judgement in the Puttaswamy case
which has made privacy a fundamental right. Linking the voter ID with Aadhaar violates the right to privacy as
defined by the judgement, and the competence of the government to legislate on the matter has been questioned.
The Supreme Court judgement on Aadhaar had limited its use to welfare schemes, whereas voting is a
constitutional right. The use of Aadhaar in matters related to citizenship might go beyond its scope as prescribed
by the court.
107. Out of the following, which one provides strength to the author’s arguments?
(a) The members of the Standing Committee of Law and Personnel are merely the puppets of the ruling party.
(b) A detailed investigation and analysis of each of the provisions of the Electoral Bill was done before passing
it.
(c) The government called for the public discussion of the Aadhar Bill but the general public did not pay heed.
(d) All of the above provides strength to the author’s arguments.
Head Office: 127, Zone II, MP Nagar, Bhopal |+91-7676564400| https://www.toprankers.com Page 27 of 36
109. Which option weakens the author’s arguments the most?
(a) Reforms in the electoral matters of India were long overdue.
(b) Linking Aadhaar with Voter ID will help weed out fraudulent voters.
(c) Linking Aadhaar with Voter ID will increase the risk of multiple enrollments.
(d) The possibility of disenfranchisement of voters with the help of the new law is a possibility
110. Which of the following must be true for the author’s arguments in the passage to hold ground?
(a) There is no need to make a law on something that is optional.
(b) If right to property was not a fundamental right, then this bill would have little room for criticism.
(c) If a bill passes the scrutiny of the parliamentary panel, then all the concerns of the bill can be ignored.
(d) Linking the voter ID with Aadhaar gives the government transparency into nefarious activities by criminal
elements.
Passage(Q.111-Q.115): Results of the exercise carried out by the J&K Delimitation Commission are in the
public domain even though the proposals haven’t been published in the gazette. Jammu is to get six more seats
in the proposed assembly and Kashmir will be given one more. It will lead to a split of 43 seats for Jammu and
47 to Kashmir in a 90-member assembly. Kashmir’s politicians, rarely on the same page, are unanimous that
their region got a raw deal. They still have time to convince the Commission. The J&K Delimitation
Commission is a unique body, owing its existence to the provisions of the Jammu and Kashmir Reorganisation
Act, 2019. The Act set the terms for the Commission. In addition to population, other aspects such as physical
features, communication and conveniences were to be considered in the delimitation exercise. To put the
extraordinary situation of J&K in perspective, it’s important to consider Andhra Pradesh’s bifurcation. The 2014
AP reorganisation law states that the assembly seats in the successor states will be increased. That exercise has
been put off till the first census after 2026, which is when there can be a national delimitation exercise.
Chief election commissioner Sushil Chandra, a member of the J&K Delimitation Commission, observed that it’s
not a mathematical exercise, but must reflect political aspirations. That’s the nub of the matter. It’s the reason
the Commission now needs to engage politicians from Kashmir. Advancing the delimitation exercise for just
J&K serves no purpose if it cripples a nascent political process. Reactions to this exercise should also make us
rethink the forthcoming national delimitation exercise. The Constitution initially linked Lok Sabha seats allotted
to a state to its population. Subsequent amendments froze the population reference point to the 1971 Census till
the first census after 2026. It’s meant to protect states that took a lead in lowering fertility rates.
A delimitation exercise represents a shift in political power. As population is the main basis of seat allocation,
an inherent problem is it dilutes the political clout of states and regions that have done well in realising the goal
of population control. India has over time overcome fissiparous tendencies that weakened national integrity.
Given this backdrop, a national delimitation exercise is best avoided because of the danger of unintended
consequences. And in J&K, the Commission should find a way to address the fears of Kashmir before it publishes
its outcome.
111. Out of the following, which one is not one of the supporting ideas of the passage?
(a) J&K Delimitation Commission should address fears of Kashmiri people.
(b) India should rethink about the 2026 national delimitation plan.
(c) Indian national integrity has weakened to a new low.
(d) Advancing the delimitation exercise for J&K should be mindful of budding and new political process.
112. With which of the following is the author most likely to agree with?
(a) The intention of the recent Delimitation exercise in J&K may be to fuel communal disharmony.
(b) The ruling party is setting double standards for the north and south of India.
(c) The Delimitation Exercise in J&K is a unique exercise for several reasons.
(d) Giving population importance in a delimitation exercise is beneficial to national integrity.
Head Office: 127, Zone II, MP Nagar, Bhopal |+91-7676564400| https://www.toprankers.com Page 28 of 36
113. It can be inferred from the passage that:
(a) Delimitation exercise is one of the features of a democratic society.
(b) Imposing heavy handed measures in Kashmir is not sustainable.
(c) A national delimitation exercise may weaken national integrity.
(d) The govt. should focus on guaranteeing jobs instead of making illicit moves.
114. Consider the following statement: “The J&K Delimitation Commission has invited the views of the public and
political parties until the end of next month.” What role does this statement play towards the passage?
(a) It weakens the author’s arguments.
(b) The author is likely to welcome this move.
(c) It does not affect the author’s arguments.
(d) It is an inference that can be drawn from the author’s arguments.
115. The author cites Andhra Pradesh’s bifurcation to
(a) Drawn an analogy between Jammu and Kashmir and Andhra Pradesh’s situation.
(b) To highlight the discrepancy in exercise of the J&K Delimitation Commission.
(c) To mention the contradiction between Jammu and Kashmir and Andhra Pradesh’s situation.
(d) To challenge the decision taken by the J&K Delimitation Commission.
Passage(Q.116-Q.120): The sweeping victory of Gabriel Boric, in the presidential run-off election, is a
testimony to how Chile has changed. One of the bastions of free market orthodoxy in Latin America, Chile has
been rocked by anti-inequality protests for more than two years. Mr. Boric, one of the protest leaders who
promised to “bury neoliberalism” during his campaign, built an alliance of social democrats and communists
that took on the Republican Party’s José Kast. While Mr. Boric promised to build a more equitable society in
one of the most unequal countries, Mr. Kast, a defender of the military regime, positioned himself as a candidate
of the economic status quo and blamed migrants, terrorists and narco-traffickers for Chile’s agonies. The
pollsters had predicted a narrow lead for Mr. Boric, but his 12-point triumph over Mr. Kast by securing about
56% of the votes marked the strongest political comeback of the Chilean left, which had undergone systemic
persecution during the U.S.-backed military dictatorship of General Augusto Pinochet. Gen. Pinochet, who
toppled the socialist President, Salvador Allende, in 1973, laid the foundations of Chile’s neoliberal state. His
regime fell in 1990, but the state apparatus he built survived, including the Constitution. Now, when an elected
Constituent Assembly is writing a new Constitution for Chile, bringing an end to Pinochet’s influence, the
country will have the most left-wing President since Allende.
Mr. Boric has promised to fight the “privilege of the few” and tackle poverty and inequality. He has opposed
big-ticket mining projects as part of his climate protection plan. He wants to raise taxes by 8% of GDP, abolish
the unpopular private pension funds, shorten the working week to 40 hours, raise the minimum wage and create
a universal health-care system. These promises were the crux of the progressive electoral platform he built. Mr.
Boric, who will be sworn in next month, faces the daunting challenge of walking the talk. His legislation agenda
would be met with strong opposition in Parliament: the Senate is evenly split between the right and the left, and
in the 155-member Chamber of Deputies, his coalition has only 37 MPs. Sagging growth and high inflation
would limit the new government’s spending agenda. If he goes ahead with the plan to raise taxes on the
corporations, abolish private pensions and waive off student debt, the private capital and the old political
establishment would revolt, like what happened in the other left-ruled states in Latin America. Mr. Boric’s
victory has put wind in the sails of Chile’s left-wing politics, but he should be ready for a storm as he seeks to
take on the Pinochet consensus.
118. Which among the following can be inferred from the passage above?
(a) Mr. Boric is contradicting himself by banning big mining projects if he wants a more equal society.
(b) Through the election, it is clear that Chile wants to elect conservative politicians.
(c) Controlling inflation would help the Boric government fulfill its promises.
(d) José Kast represented the Democratic Party.
119. The author has made a few claims in the passage. For which of the following has the author presented a few
pieces of evidence to support his claims?
I. The Boric government would face stiff opposition in the Parliament.
II. It would be difficult for Boric to fulfill his ambitious promises.
III. The Chilean left had undergone systemic persecution under Pinochet.
(a) Both I and II (b) Both I and III (c) Both II and III (d) All I, II, and III
120. What challenges is the Boric government not likely to face in its tenure?
(a) Active opposition. (b) Soaring growth rate.
(c) Protests from the people. (d) An easy tenure.
Passage(Q.121-Q.125): According to Jewish and Catholic traditions, human beings are the most advanced living
organisms on earth, in part because of their capacity to speak and to think creatively. While animals such as
nonhuman primates or human-animal chimeras may exhibit human-like cognitive capacities, they will still be
considered nonhuman animals according to both traditions. In contrast, personhood status and rights reflect
society’s perception and understanding of whether an organism possesses human-like cognitive capacities. Thus,
society has the right to use its moral compass to decide what animals should possess personhood status.
Another consideration for granting personhood status is the definition of “life.” Jewish thought maintains that
only a living organism can attain personhood. This criterion conflicts with the decision of Saudi Arabian
government officials, who recently became the first to grant full citizenship to a robot, the artificially intelligent
Sophia, developed by Hanson Robotics.
Biotechnologies can elicit complex ethical challenges that need to be discussed and addressed. I believe that an
animal that has been engineered with human brain cells or human neural organoids does not necessarily attain a
human state. Being human requires the creation of an embryo from human gametes or being born from a human.
However, a human-animal chimera that expresses basic human-based cognitive capacities or human-like
behaviors (that is, advanced communication skills, the ability to override basic instincts, etc.) can be granted
personhood status reflected in legal or moral rights not normally given to animals. For example, society can
confer these organisms with limited autonomy and prohibit their use in clinical drug trials based on our
perception of their ability to provide consent. As with other medical ethical dilemmas, personhood status of any
living organism must be assessed on a case-by-case basis.
121. The passage is mainly concerned with which of the following?
(a) Considerations used as moral compass for granting personhood to human-animal chimera.
(b) Features and characteristics of human-animal chimeras.
(c) Ways in which human-animal chimeras have been granted personhood.
(d) Problems in granting personhood to the closest relatives of human beings.
Head Office: 127, Zone II, MP Nagar, Bhopal |+91-7676564400| https://www.toprankers.com Page 30 of 36
122. Which among the following techniques has the author used to make arguments in this passage?
(a) The author tries to bring in different perspectives about a question before giving more credibility to a
particular perspective.
(b) The author employs arguments based on cause and effects.
(c) The author asserts arguments of different authors to strengthen his point of view.
(d) The author brings forth various perspectives and concludes by asserting his point of view concerning the
subject matter.
123. According to the author, which of the following is the most important consideration before granting nonhumans
personhood?
(a) Human-like cognitive abilities. (b) Moral and legal ethics.
(c) Advanced communication skills. (d) None of the above.
124. Based on the passage, which of the following is most likely to be the profession of the author?
(a) A professor of Biology. (b) A political scientist.
(c) Author of a science-fiction. (d) A politician
125. What is the main purpose of the author behind introducing Jewish and Catholic traditions?
(a) The author is trying to find some points of difference between the two traditions.
(b) The author advocates Jewish traditions to grant human-animal chimeras’ personhood.
(c) The author tries to bring in different perspectives about granting human-animal chimeras’ personhood.
(d) The author is trying to explain how Jewish and Catholic traditions are similar in granting something
personhood.
Passage(Q.126-Q.130): The world is familiar with the cycles of boom and bust in the energy market. It would
be understandable, therefore, if the current surge in the price of natural gas in Europe and its rippled impact on
the price of coal and oil is seen as just one more twist in the sector’s business trajectory. That would be a mistake.
For, this time, there is the added dimension of its consequence on the pace, nature and course of the “green
transition”. Were this dimension ignored, the price hike would incentivize an increase in the production of fossil
fuels and that would run counter to not just public sentiment but also the efforts to shift to a clean non-fossil fuel
energy system. The nub of the dilemma for governments created by this latest price shock is to find a way to
navigate the long-term imperatives of decarbonisation and also manage the political and social backlash from
consumers impacted by high electricity and fuel costs. COP 26 should add the resolution of this dilemma to its
agenda. It will be a recurring issue. The price of natural gas in Europe has shot up by approximately 600 per cent
over the past 12 months. And, in a reversal of the conventional feedback loop wherein the price of oil would lead
to a change in the price of gas, this time it is the price of gas that has pushed up the prices of oil and coal. This
is because as the former became increasingly unaffordable, consumers turned to the latter. There are many
reasons for this price surge, but they distill down to the perennial influencers of the energy market — the interplay
of demand, supply and geopolitics. On the demand side, the strongest driver has been the global economic
recovery. Added to that are the micro factors of the drop in hydropower in Brazil and China because of drought,
the reduction in wind power because of unfavourable wind conditions in the North Sea and the underperformance
of nuclear reactors in Europe. On the supply side, there have been three economic blockers and one geopolitical
bottleneck. The economic blockers were the cold wave in Texas in February this year, which froze gas wells and
throttled the export of US LNG, the start-up of the maintenance work suspended since 2020 on account of Covid-
19. Matters have been compounded by the diversion of US LNG cargoes destined for Europe to Asia and low
inventories.
126. According to the given passage, which of the following statements can be attributed to the author?
(a) The price of oil and the price of gas are remotely connected.
(b) The reasons for the price surge of gas are alien to what we have seen before.
(c) This price surge of gas is unlikely to be a one-off event.
(d) The most important factor of the deflation of the price of gas is yet to be ascertained.
Head Office: 127, Zone II, MP Nagar, Bhopal |+91-7676564400| https://www.toprankers.com Page 31 of 36
127. Out of the following, which among the following is not one of the supporting factors of the rise in prices of oil?
I. The global economic recovery is one of the strongest reasons for the given event.
II. The cold wave in Texas was an enabling factor.
III. The drought in China was an important cause for the price surge.
(a) Only I (b) Only I and II (c) All I, II, and III (d) None of the above.
128. Which of the following can be validly inferred from the given passage?
(a) The current surge in the price of gas is unlikely to have been perceived as an abnormal event.
(b) The recovery of the global economies is the most important factor in the increase in the price of gas.
(c) The price of natural gas in Europe has shot up by approximately 600 per cent over the past 12 months..
(d) The price of gas is inversely proportional to the price of oil.
129. Out of the following, which one is not true from the passage?
I. If the price of gas in Europe one year ago was Rs. 100 per litre, it would have become Rs. 600 per litre.
II. Consumers have turned to natural gas since it is perceived that fossil fuels are a major source of pollution.
III. The governments have a task cut out for them as far as managing the backlash of the people due to the rise
in the price of gas is concerned.
(a) Only II (b) Only III (c) Only I and II (d) I, II & III
130. Which among the following is not one of the arguments of the author?
(a) The current surge in the price of gas is different from previous such events.
(b) The major contributors to this price surge are not entirely different from previous causes.
(c) The price hike of gas would lead people to consume more fossil fuels.
(d) On the demand side, the strongest driver has been the global economic recovery.
131. Eight persons are sitting around a circular table facing inward. B is sitting second to the right of O. U is sitting
third to the left of A. N is sitting to the immediate left of J. N is sitting opposite to R. M is sitting to the immediate
right of B. Who is sitting second to the left of R?
(a) J (b) U (c) A (d) M
132. In the following question, two statements are given and three conclusions are given below these statements.
Choose the option which shows the conclusions which logically follow from the given statements, disregarding
commonly known facts.
Statements:
Only a few leashes are ropes.
Only a few ropes are cords.
Conclusions:
I. All ropes are leashes.
II. Some leashes can be cords.
III. Some ropes are not cords.
(a) Only I and II follow (b) Only II follow
(c) Only I follow (d) Only III and II follow
Head Office: 127, Zone II, MP Nagar, Bhopal |+91-7676564400| https://www.toprankers.com Page 32 of 36
133. In the question below, two statements are given followed by conclusions: I, II and III. You have to take the given
statements to be true even if they seem to be at variance from commonly known facts. Read the conclusions and
decide which of the given conclusions logically follows from the given statements disregarding commonly
known facts.
Statements:
All hair are black.
All black are eyes.
Conclusions:
I. No hair is an eye.
II. All hair are eyes.
III. Some eyes are black.
(a) Either I or II follows (b) Only II and III follows
(c) Only II follows (d) Only III follows
134. Select the letter cluster that can replace the question mark (?) in the following series.
PROCASTINATION, PSOCASTINATIPN, PSPCASTINATJPN, PSPDASTINAUJPN, ?
(a) PSPDBSTINBUJPN (b) PSPDBSTINAUJPN
(c) PSPDBTUINATJPN (d) PSPDBTUINAUJPN
135. A boy started walking towards north and travelled 2 km before he turned right. He then travelled 3kms further.
Then he repeated the process of taking left and travelling 3km and then taking right and travelling 4km, two
times. What is the shortest distance between his current position and his starting point?
(a) √117 km (b) √231 km (c) √123 km (d) √185 km
Head Office: 127, Zone II, MP Nagar, Bhopal |+91-7676564400| https://www.toprankers.com Page 33 of 36
the defendant’s conduct. However, no team player deduced from the passage. Option (a) is incorrect as
has consented to partake or consent to the harm such an altercation can not be ascertained by any
caused by any other player which is outside the reasonable person but is also not consented to due to
match and is a result of physical altercation. similar reasons. Option (b) is incorrect because the
Therefore, Babur cannot be excused from the question of contributory negligence has not been
liability on the premise of implied consent and under discussion according to the present factual
would thus be liable. Option (b) is incorrect because matrix but the question pertains about whether a
any altercation cannot be called to be a part of the consent has been given to succumb the harm caused
sports itself as the plaintiff (in this case the during the match due to physical altercation, which
teammate), has not consented to such a behaviour of due to reasons stated above is in negative. Option (c)
the teammate so as to mitigate the liability and is incorrect as the passage has clearly provided that
therefore the option is incorrect. Option (c) is consent can also be implied and thus the need for
incorrect as the option fails to draw any logical consent to be in written form is incorrect.
conclusion between the passage and the factual
matrix in hand as despite the fact that if the fight SECTION - D : LOGICAL REASONING
occurred within the stadium, would Babur be
exempted from liability due to implied consent. 106. (a) The correct answer is A. The author in the passage
Option (d) is incorrect as it fails to draw any logical is primarily concerned with the fact that the bill was
and legal explanation to justify the situation in hand. passed without any debate or discussion. It is
104. (b) Correct answer is Option (b) as, there is no consent important to understand that the author in the
given by the audience present in the stadium for the passage does not consider the bill to be progressive
harm caused due to the negligence of the stadium or carrying any positive provisions at all and hence,
authorities and hence stadium authorities can be both Options B and C are incorrect. Option D is
made liable. Kindly note that the operative term in incorrect since it is only one of the supporting ideas
the passage is the participation in the defendant’s of the passage and cannot be considered as the main
conduct. The defendant in the present case, being idea of the passage due to its limited scope.
the stadium authorities, there was no participation or 107. (a) The correct answer is A. The author is not satisfied
consent of any such participation where there is an with the response of the Law Minister. This option
agreement between the parties to endure any loss or provides strength to his argument that the response
harm due to the conduct of the other party and hence of the Law Minister is not enough. Both Options B
the stadium authorities can be made liable. Option and C are incorrect since they weaken the author’s
(a) is incorrect as the question is not only restricted position. Option D is incorrect by virtue of options
to the negligence (whether apparent or not) of the B and C being incorrect.
stadium authorities but whether a consent has been 108. (d) The correct answer is D. The author feels that the
given by the audience members for all the harm bill can lead to a situation where Electoral
caused during the match/ inside the stadium or not, registration officers can demand the Aadhaar
the categorical answer to which would be in number of those who seek a voter ID and even of
negative. Option (c) is incorrect as consent is those who already have the card. Therefore, he feels
effective only till the time it signifies participation that the line between mandatory and voluntary
in the defendant’s conduct and in the absence of linking may be breached. Option A is incorrect since
such a consent no such benefit can be provided to the author in the last paragraph criticizes the bill on
the perpetrator. Option (d) is incorrect because it the basis of previous SC judgements. Option B is
would be incorrect to state that once a person enters incorrect since the author is not concerned about the
the premises, s/he waives all protection against harm right to property in this case. Option C is incorrect
for having consented to such harm due to the since it cannot be answered with the use of
operative parts stated above along with other information in the given passage.
exceptions where the consent cannot be treated as 109. (b) The correct answer is B. The author in the entire
valid. passage talks about the concerns related to the
105. (d) Correct answer is Option (d) as there is no consent electoral bill. However, this option presents a
given by the opponent team supporter for such a positive side of the bill and hence weakens the
harm caused due to any physical altercation as it is author’s argument. Option A may seem close
not a part of the usual course of the match. As has however, the author never denies that reforms in the
already been pointed out that in order for the defence electoral matters of India were overdue. Option C is
of consent being available, such a consent has to be incorrect since it in fact strengthens the author’s
for the defendant’s conduct. However, by buying the arguments. Option D strengthens the author’s
ticket of a stadium a person doesn’t consents to the argument.
harm which might be endured to physical altercation 110. (a) The correct answer is A. This option must be true
between the spectators themselves and hence no for the author’s argument to hold. If this was
consent whether expressed or implied can be incorrect, the author would not have criticized the
Head Office: 127, Zone II, MP Nagar, Bhopal |+91-7676564400| https://www.toprankers.com Page 11 of 16
bill on the ground that it was optional. Option B is as the above statement corroborates ideas of the
incorrect since the passage is not concerned with the author and it does affect the author’s argument is a
right to property. Option C is incorrect to be a must- positive manner. Option D is incorrect, as it is not
be-true option due to the fact that it is an extreme an inference that can be taken out from the passage.
option. There is nothing in the passage to suggest An inference requires an explicit mention of a fact
that all the concerns of the bill can be ignored for the before, which is not the case here. The main
reason that it has passed the scrutiny of a statement is more or less the summation of what the
parliamentary panel. Option D cannot be true for author believes in.
author’s argument to hold true. 115. (a) Option A is the correct answer. The mention of
111. (c) The correct answer is C. It is correct since the author Andhra Pradesh is with respect to the exercise by the
does not indicate that even at present, India’s J&K Delimitation Commission. The author cites the
integrity is low. Option A is incorrect since the example of Andhra Pradesh to draw similarity
author makes this argument throughout the passage between the two states with respect to putting the
and also re-makes this in the last paragraph. Option extraordinary situation of J&K in perspective and to
B is also incorrect since the author warns about the highlight that such a move has been That exercise
dangers of such an exercise. Option D finds a has been put off till the first census after 2026, which
mention in the passage and is supported by the is when there can be a national delimitation exercise.
author. Refer to the line, ‘Advancing the Option B is incorrect as there is no discrepancy that
delimitation exercise for just J&K serves no purpose the author mentions with regard to AP’s bifurcation.
if it cripples a nascent political process.’ Options C and D are irrelevant, as neither the author
112. (c) The correct answer is C. Refer to the lines, ‘The uses the Andhra Pradesh’s bifurcation to contradict
J&K Delimitation Commission is a unique body, or challenge the existing situation.
owing its existence to the provisions of the Jammu 116. (a) The correct answer is A. The author has considered
and Kashmir Reorganisation Act, 2019.’ If The J&K his victory as a significant event in Chilean politics
Delimitation Commission is a unique body, it means and has also argued throughout the passage that it
that certain decisions taken by the Commission would be a daunting task for him to walk the talk.
would be unique too. The author is unlikely to agree Therefore, this is the most appropriate main idea of
with Option A since there is no such indication in the passage. B is incorrect since the author does not
the passage. Option B is also incorrect since as it make such a claim in the passage. C is also an
doesn’t correspond with the information in the extreme option. The author never claims that left
passage. Option D is also incorrect since in the last wing politics is best for Chile. D is appealing
paragraph, the author argues that such a move has however the victory of Mr. Boric was never easy.
weakened national integrity in the past indicating That is the problem with this option. He may have
that it may affect national integrity in the future as had a sweeping win, but to say that it was easy
well. The author would most likely agree with victory is opposite to what has been stated in the
Option C. passage as the comeback of the left has not been
113. (c) The correct answer is C. The author warns about the easy.
dangers of the unintended consequences of a 117. (d) The correct answer is D. None of the options are
national delimitation exercise just after speaking true. Both A and B are incorrect since the author has
about the fact that giving population high only said that it would be a daunting task for Mr.
importance in a delimitation exercise may weaken Boric to fulfill his promises - we cannot conclude
national integrity. A close reading of the last either A or B from here. C, on the other hand, is
paragraph makes this the correct answer. A is incorrect due to a factual error - Pinochet was
incorrect since the author has not established any against the left-wing.
link between these two. Option B is incorrect as well
since no such indication has been given in the 118. (c) The correct answer is C. The author says that Mr.
passage. The author does not claim that delimitation Boric would face challenges in the form of high
exercise is an illicit move, hence Option D is inflation. Therefore, it is reasonable to infer that
incorrect as well. controlling inflation would be beneficial. Option A
114. (b) The correct answer is B. There is a very thin line is incorrect since Mr. Boric wants a more equitable
between what strengthens the author’s claims and society and for that he would ban big mining
what move will be welcomed by the author. The projects. There is nothing contradictory about this.
author has argued that the commission should Option B is incorrect since Chile has elected a
consider the views of the people concerned. progressive politician (Mr. Boric). Option D is
Therefore, such a move will be welcomed by the incorrect, as José Kast represented the Republican
author and hence Option B is the correct answer. Party.
Option A is ruled out, as the statement does not 119. (a) The correct answer is A. (i) is clearly correct. The
negate the contention of the author; therefore, author presents the data in the subsequent sentence
cannot weaken the argument. Option C is incorrect, of making this claim. Refer to the lines, ‘the Senate
Head Office: 127, Zone II, MP Nagar, Bhopal |+91-7676564400| https://www.toprankers.com Page 12 of 16
is evenly split between the right and the left, and in passage and hence B is incorrect. C is incorrect since
the 155-member Chamber of Deputies, his coalition there is no indication that the question about
has only 37 MPs.’ The author has already provided personhood does not need any answers. D is correct.
pieces of evidence for his claim in (ii) by talking The author brings in various perspectives on human-
about stiff opposition, sagging growth rate, high like chimeras and their personhood.
inflation, etc. (iii) is incorrect since there is no 123. (d) The correct answer is D. This is a simple question.
supporting evidence for this claim. The rest of the The last line of the passage makes it clear that there
options are ruled out by virtue of option A being the is no consideration that is the most important in
correct answer. granting personhood to non-humans. It depends on
120. (d) The correct answer is D. A is wrong - the opposition case-to-case basis. Refer to the lines, ‘The author
is active as per the passage hence he will face it. The brings in various perspectives on human-like
growth rate of Chile is sagging as opposed to chimeras and their personhood.’
soaring, making option B as the incorrect answer. 124. (a) The correct answer is A. The author in the passage
The passage mentions that the he had a sweeping brings in different perspectives on a biology
victory which means that he had the support of the question. Also, the language suggests someone with
public; therefore, there is likelihood that people will in-depth knowledge on the subject. Therefore, he is
support his decisions and not protest. Option D is most likely to be a professor of biology. B, C and D
something he is not likely to face. The last are irrelevant.
paragraph clearly states that his tenure will not be 125. (c) The correct answer is C. Clearly, the purpose of
easy, making option D correct. An easy tenure is one bringing in the perspective of Jewish and Catholic
thing he is not likely to face. traditions is to bring in different perspectives on the
121. (a) The correct answer is A. The author in the passage central idea of the passage. The main purpose of the
mainly talks about the various considerations that author is not to find similarities or differences
need to be taken into account before granting living between the two traditions. Therefore, both A and D
organisms (in particular human-animal chimera) are incorrect. Also, the author does not advocate for
personhood. Option B is far-fetched and not relevant any one particular perspective hence B is incorrect.
to the passage. Only Option C seems close; 126. (c) The correct answer is (c). The author has clearly
however, the author does not give any evidence mentioned that this price shock would be a recurring
about how such chimeras have been granted event. Hence, it can be inferred that this price surge
personhood. Option D is incorrect since the author is unlikely to be a one-off event, but a recurring one.
talks about human-animal chimeras. All the other options are easy to eliminate. Option
122. (d) The correct answer is D. The author brings in (a): The author has clearly established a connection
various perspectives on human-like chimeras and between the price of gas and oil. Option (b): The
their personhood, and in the end presents his point reasons for this price shock have been not too
of view. Option A is incorrect, as though the different - they distil down to the perennial
question is discussed in the passage whether influencers of the energy market. Option (d): There
personhood should be granted to human animal like was no price deflation - there was inflation!
Chimera, the author does give more credibility to 127. (d) The correct answer is (d). This was a trick question
any one perspective, but gives his own version of because the passage has reasons for the surge in
personhood. Refer to the lines, ‘I believe that an prices of natural gas and this question talks of oil.
animal that has been engineered with human brain Hence, option (d) is correct.
cells or human neural organoids does not necessarily 128. (a) The correct answer is (a). The author has mentioned
attain a human state. Being human requires the in the beginning of the passage that it is
creation of an embryo from human gametes or being understandable that the current surge in the price
born from a human. However, a human-animal could lead people to perceive that this is a normal
chimera that expresses basic human-based cognitive event. Refer to the lines, ‘The world is familiar with
capacities or human-like behaviors (that is, the cycles of boom and bust in the energy market. It
advanced communication skills, the ability to would be understandable, therefore, if the current
override basic instincts, etc.) can be granted surge in the price of natural gas in Europe and its
personhood status reflected in legal or moral rights rippled impact on the price of coal and oil is seen as
not normally given to animals.’ Option B is just one more twist in the sector’s business
incorrect, as the passages does not discuss a cause trajectory.’ Option (b) is incorrect because the
and its effect. Option C is incorrect, as there are no author suggests that the perennial factors play the
different authors involved. The various references most important role. Option (c) is not an inference,
are with respect to perceptions of what constitutes as it is a direct extraction from the passage. Option
personhood. (d) is incorrect, as it is contradicting the information
The author does not heed himself to any particular in the passage. Refer to the lines, ‘this time it is the
more credibility hence A is incorrect. Human-like price of gas that has pushed up the prices of oil and
chimeras and personhood is the central theme of the coal. This is because as the former became
Head Office: 127, Zone II, MP Nagar, Bhopal |+91-7676564400| https://www.toprankers.com Page 13 of 16
increasingly unaffordable, consumers turned to the
latter.’ From the statement, it is clear that increasing
unaffordability of gas pushed up the demand for oil
and coal which resulted in their price hike. There is
a direct relationship between the two.
129. (c) The correct answer is (c). [I] is incorrect because the
price of gas in Europe has shot up by 600 per cent
and not to 600 per cent. This means that if the price
was Rs. 100 per litre, it would rise by Rs. 600 and
become Rs. 700 per litre. II: While it is true that the
A is sitting second to the left of R.
consumers have turned to natural gas, this is not
132. (d) The given statements are,
because it is perceived that fossil fuels are major
Only a few leashes are ropes. Only a few ropes are
pollutants but since their prices have increased. III
cords.
is true and is mentioned in the given passage. Refer
The basic minimum overlapping diagram and
to “The nub of the dilemma for governments created
another possible diagrams are,
by this latest price shock is to find a way to navigate
the long-term imperatives of decarbonisation and
also manage the political and social backlash from
consumers impacted by high electricity and fuel
costs.”
130. (c) The correct answer is (c) since the author has not
made this argument. In fact, he claims that this price
surge is different from the rest and is not likely to
lead people to consume more fossil fuels. Option (a)
is incorrect because the author claims this in the first Here, all ropes are not leashes, some leashes can be
part of the passage itself - an easy option to cords and some ropes are not cords.
eliminate. Option (b) is incorrect because the author Hence, conclusions II and III follow and conclusion
has mentioned this when he talks about the reasons I does not follow
for this price increase. Option (d) is incorrect, as it 133. (b) All hair are black. All black are eyes.
is explicitly mentioned in the passage.
131. (c) N is sitting to the immediate left of J. N is sitting
opposite to R.
All hair are eyes. Some eyes are black. So, both
conclusions II and III follow.
134. (a) PROCASTINATION
R+1=S
O+1=P
B is sitting second to the right of O. M is sitting to PSOCASTINATIPN
the immediate right of B. O+1=P
I+1=J
PSPCASTINATJPN
C+1=D
T+1=U
PSPDASTINAUJPN
A+1=B
A+1=B
PSPDBSTINBUJPN
135. (d) The movement is shown in the image:
U is sitting third to the left of A.
Head Office: 127, Zone II, MP Nagar, Bhopal |+91-7676564400| https://www.toprankers.com Page 14 of 16
= (4C2 × 11C1 + 4C3 )/15C3
= (66 + 4) × 6/ (15 × 14 × 13)
= 70 ∗ 6/ (15 × 14 × 13)
2
=
13
139. (d) Probability of picking no red balls
= 10C3 /13C3
120
=
Minimum distance=(121+64) 1/2 = √185 286
So, Probability of at least two red balls
120
SECTION - E : QUANTITATIVE TECHNIQUES = 1−
286
136. (a) The probability that all the three balls are of same 166
=
286
color 83
= (4C3 + 6C3 + 3C3 )/15C3 =
143
= (4 + 20 + 1) × 6/15 × 14 × 13 140. (d) The probability that all the balls are of different
= 25 × 6/15 × 14 × 13 colors
5 = (2𝐶1 ) × (4𝐶1 ) × (6𝐶1 ) × (3𝐶1 )/15𝐶4
=
91 = (2 × 4 × 6 × 3 × 4 × 3 × 2)/(15 × 14 × 13 ×
137. (a) Probability of both the balls of white color
12)
= 4𝐶2 /13𝐶2 48
=
= 6/ 78 455
= 1/13
138. (b) The probability that at least two red balls are drawn
141. (c) Number of male and female form Germany who Percentage more
visited Taj Mahal = 720 + 1400 = 2120 = (9600 − 240)/240 × 100 = 3900%
142. (b) Required ratio = 710:1440 = 71:144 148. (a) Average number of vehicles through Plaza =
143. (d) Average number of Female from USA, Japan and (12240 + 720 + 540 + 9600 + 240)/5 =
Germany = (1000 + 1200 + 1400)/3 = 1200 4668
144. (a) Required % = 2800/2640 × 100 = 106.06% Only Bike and Truck number are more then 4668.
145. (a) Required difference = 2640 – 1710 = 930 So 2 Types.
146. (b) Revenue obtained will be from cars only. 149. (d) Trailer charge = 200
= 720 × 40 Total amount = 200×240 = 48000
= 28800 Car charged = 12
147. (c) No of Trucks = 9600 Total amount = 12×720 = 8640
No of Trailers = 240 Difference = 48000 – 8640
Head Office: 127, Zone II, MP Nagar, Bhopal |+91-7676564400| https://www.toprankers.com Page 15 of 16